You are on page 1of 76

NURSING PRACTICE I

Situation - Public Health Nurse Zafra is preparing for a meeting to discuss with his co-
workers major chronic disease which are responsible for deaths in their city. He plans
to focus on promotive and preventive health care.

1. He wants to be sure that health workers have common understanding of chronic


disease. The BEST definition of a chronic decease is:

A. problem that are experienced by individuals in their life time that may cause
financial hardships.
B. problems in body functions and structure which is a lifetime debilitation.
C. phenomenon resulting from the interaction between people and their environment.
D. a condition that requires ongoing medical care, limits one can do which is likely to
last three months or more.

2. Nurse Zafra would like to propose a month-long series of health information related
to healthy living. Some of the relevant topics should include:

1. Diet
2. Exercise
3. Pharmacotherapy
4. End-of –life care
5. Vaccination for selected condition

A. 1, 2 and 3
B. 2, 3 and 5
C. 1, 2 and 4
D. 1, 2 and 5

3. Nurse Zafra emphasizes the responsibility of the nurse to increase the knowledge of
the community regarding the importance of physical activity. The minimum amount of
daily physical activity required for health benefits is:

A. 15 minutes, to be achieved every morning or afternoon, twice a week.


B. At least 30 minutes, cumulative, of moderate intensity on most days of the week.
C. 60 minutes of intensive activity, once a week.
D. 30 minutes of aerobic activities sporadically during the week

4. Nurse Zafra explains the ‘ABC’ strategies to promote healthy nutrition. These ‘ABC’
strategies are the following EXCEPT:

A. build healthy nutrition-related practices


B. aim for ideal body weight
C. choose food wisely
D. access weight-reducing herbs

5. Stress management is also included in the proposed program. Meditation is one of


the several stress management techniques which can have beneficial effects. Which of
the following effects is NOT included?

A. Helps muscles relax


B. Eliminates stressful thoughts
C. Lower blood pressure
D. Increase respiratory rate

Situation – The public health nurse must have an understanding of the demography
which should support health care plan.

6. Demography is concerned with the study of population. Which of the following are
included in a demographic profile?

A. Size, composition health status, and environment


B. Change in population, distribution, and health status
C. Size, distribution, composition, and change in population
D. Size, distribution, and composition
7. Based on 2017 Demographic Report (July 2016 estimate), the Philippine population
has reached:

A. 140 million
B. 120 million
C. 102 million
D. 85 million

8. There are three demographic variables in population. Which one is NOT included?

A. Fertility
B. Migration
C. Morbidity
D. Mortality

9. Population structure is a diagram of population typically presented in a pyramid-like


style format based on __________.

A. sex and education attainment


B. age and fertility
C. age and civil status
D. age and sex

10. An aging population refers to a phenomenon in which the median age of the
population in a county rises significantly compared to the total population. This is
caused by any or combination of the following:

1. A declining birth rate


2. Rising life expectancy
3. Decreased fertility
4. Increased child survival
5. Better health

A. 1, 2, 3, and 4
B. 1, 2, 4, and 5
C. 1, 2, and 3
D. 1, 2, 3, 4, and 5

Situation – Nurse Lunario makes several home visits to patients who were discharged
from a district hospital.

11. As Nurse Lunario prepares for the day’s visits,which of the following patients should
be seen FIRST? A client with __________.

A. Parkinson’s disease with imbalance gait


B. A history of congestive heart failure with dyspnea
C. a thoracostomy month ago
D. a stroke with nasogastric tube (NGT)

12. Mang Bonifacio refuses to take his daily medication for hypertension. Which of the
following actions should Nurse Lunario take at this time?

A. Tell Mang Bonifacio that he will suffer from stroke


B. Explore the reason for the client’s refusal to take the medication
C. Administer the medication by injection
D. Obtain help from relatives in administering the medication

13. Nurse Lunario sees a 65 year-old male, who is recovering from stroke. He noticed
that Mang Jose exhibits signs of unilateral neglect. Which behavior is suggestive of this
unilateral neglect? Mang Jose is __________.

A. unable to carry out cognitive and motor activity at the same time
B. observed shaving only one side of his face
C. unable to complete a range of vision without turning his head side to side
D. unable to distinguish between two tactile stimuli
14. Nurse Lunario is teaching a client with Parkinson’s disease on ways to prevent
curvatures of the spine associated with the disease. To prevent spinal flexion. The nurse
should tell the client to __________.

A. rest in supine position with his head elevated


B. sleep only in dorsal recumbent position
C. periodically lie in prone position without a neck pillow
D. sleep on either side, but keel his back straight

15. A client with congestive heart failure has been recovering digoxin (Lanoxin). Which
finding indicates that the medication is having a desired effect?

A. Increased weight
B. Improve appetite
C. Increase urine output
D. Increase pedal edema

Situation - The goal of ethical reasoning in the context of nursing is to reach a mutual
peaceful agreement that is in the best interest of the client. Reaching an agreement
may require compromise.

16. When a ethical issue arises, which of the following is the most important nursing
responsibility?

A. Ensure that a team is responsible for deciding ethical questions.


B. Remain neutral and detached when making ethical questions.
C. Be able to defend the morality of one’s own actions.
D. follow the client and a family wishes exactly

17. Which of the following is a clear violation of the underlying principles associated
with professional nursing ethics?

A. a health care facility policy permits the use of internal either fetal monitoring which
in the literature evidence points to either support or refuse this practice.
B. the nurses on the unit agree to sponsor a fund raising event to support a labor strike
proposed by fellow nurses at another health care facility
C. a client reports that he didn’t quite tell the doctor the truth when asked if he was
following his therapeutic diet at home
D. when asked about the purpose of a medication, a nurse colleague respond, “Oh i
never look them up. I just give them what is prescribed”

18. Which of the following statement would be most helpful when a nurse is assisting
clients in clarifying their values?

A. “the most important thing is to follow the plan of care. Did you follow the doctor’s
order?”
B. “some people might have a different decision. What led you to make your decision?”
C. “if you had asked me, i would have given you my opinion. Now how do you feel about
your choice?”
D. “that was not a good decision. Why did you think out would work?”

19. A dying patient asks for over dosage of pills from the nurse to end his life for his
unbearable pain cause by cancer. If the nurse grants the patient’s request, she liable for

A. active euthanasia
B. negligence
C. advance directive
D. Assisted suicide

20. The midwife is verbal accusing the nurse that the latter caused the medication
errors without citing an evidence. The midwife can be sued for __________.

A. assault
B. slander
C. defamation
D. Libel
Situation – Public Health Nurse Melchora is tasked for the day to administer
vaccination to pediatric clients in the barangays she covers. She, along with the team of
midwives, should be knowledgeable on how to provide with the most effective,
necessary and safest vaccination.

21. The foundation of medication administration is the application of the “Rights of


Medication Administration”. Which of the following is NOT included in these rights?

A. Right route
B. Right patients
C. Right dosage
D. Right price

22. When conducting Parent Education on immunization, which of the following


considerations should Nurse Melchora include?

1. Welcomes and greet child and family


2. Explains the vaccines to be administered
3. Refuse entry to those who come late
4. Informs attendees about the procedures to be done
5. Provide parents time to ask questions
6. Refrain from answering questions from parents

A. 2, 4, 5, 6
B. 2, 3, 5, 6
C. 1, 2, 4, 5
D. 1, 3, 4, 6

23. When handling vaccines, the FIRST step Nurse Melchora should do is to __________.

A. select the correct needle size


B. reconstitute using the diluent supplied
C. check the content prior to drawing up
D. check the vial for expiration date

24. If more than one vaccine is given, the following guidelines should be observed,
EXCEPT _________.

A. do not give more than one dose of the same vaccine in one session
B. do not use the same arm or leg for more than one injection
C. give doses of the same vaccine at the correct intervals
D. use the same syringe and needle in administering two vaccines

25.When documenting the procedures done, which of the following should NOT be
recorded?

A. Date
B. Lot number
C. Manufacturer
D. Needle gauge

Situational – Public Health Nurse Jesusa has several elderly clients in her case load.

26. Which of the following clients is MOST likely to be a victim of elder abuse? A client
who is __________.

A. 65 year old female with hip replacement


B. 76 year old male with Parkinson’s disease
C. 80 year old female with Alzheimer’s dementia
D. 70 year old male with diabetes mellitus

27. To prevent deformities of the knee joint in an elderly client with exacerbation of
rheumatoid arthritis, which of the following instructions should Nurse Jesusa make?

A. Discourage passive range of motion because it will cause further swelling.


B. Tell the client she will need joint immobilization for three weeks.
C. Tell the client to remain on bed rest until swelling subsides.
D. Encourage range of motion of the joint within the limits of pain.

28. Nurse Jesusa is visiting a client with Paget’s disease. Which is an important part if
preventive care for the client with Paget’s disease?

A. advise the client to see the dentist regularly


B. encourage the client to take influenza vaccine
C. keep the environment free of clutter
D. tell the client take multivitamin daily

29. Mrs. Andrea is taking prescribed Aspirin, 325 mg daily, for her transient ischemic
attacks. Nurse Jesusa explains that aspirin was prescribed to _________.

A. boost coagulation
B. prevent headaches
C. prevent cerebral anoxia
D. decrease platelet aggregation

30. Nurse Jesusa is visiting a client with Stage III Alzheimer’s disease. Which of the
following is a characteristic of this stage?

A. Wandering at night
B. Irritability
C. Dementia
D. Failure to recognize familiar objects

Situation – Public health nurse participate in health screenings which help in the early
detection of a disease or identify those who are at-risk for a particular disease or
condition. Early detection, followed by treatment and management of the condition can
result in better outcomes, and lower the risk of serious complication.

31. Nurse Susana organized a health fair in one of the communities she covers, which
was well attended, with several SCREENING TEST being given simultaneously. What
would Nurse Susana consider the purpose of the health fair?

A. Community
B. Multiphasic screening
C. Community bonding
D. Case finding

32. As nursing student were setting up a health screening at a local community center,
they reviewed the need to screen for heart disease and cancer. One student asked, “But
colon cancer is prevalent; why aren’t we setting up for sigmoidoscopy?”. How would
Nurse Susana reply?

A. “Can you find adequate privacy for a sigmoidoscopy?”


B. “A sigmoidoscopy is expensive to perform and invasive”
C. “Would you like to perform that test to help arrange it?”
D. “What a great idea. Would you like to help arrange it?”

33. Sensitivity is the ability of a screening test to accurately identify. What aspects of
the screening?

A. persons with symptoms of the disease


B. persons who do not have the disease
C. persons who have the disease
D. persons who now have a diagnosis of the disease

34. The ability of the screening test to distinguish between persons with and without a
disease is known as _________.

A. reliability
B. sensitivity
C. validity
D. Specificity
35. Some common examples of screening test are the following EXCEPT:

A. Pap smear for cervical cancer screening in women


B. Cholesterol level in heart disease screening
C. PEA levels for prostate CA in men
D. Urinalysis for male and female clients

Situation – Many Filipinos are suffering from malnutrition. Nurse Aga knows that it is a
result from poor diet or lack of food.

36. Micronutrient supplementation is included in what program of the DOH?

A. Sentrong Sigla
B. Araw ng Sangkap Pinoy
C. Expanded Program on Immunization
D. Reproductive Health Bill

37. You assess for a child’s nutritional status using the mid-upper arm circumference
(MUAC). Measurement reads 120cm which falls under the orange color of a 4-colored
tape. This indicates that the child

A. should be immediately referred for supplementation


B. should be immediately referred for treatment
C. is at risk for acute malnutrition
D. well-nourished

38. You compute for the ideal weight of Edward whose actual weight is 135lbs and
height is 5’10”. His ideal body weight is ___lbs.

A. 140
B. 135
C. 154
D. 174

39. Edward’s degree of malnutrition is _____ percent of ideal body weight (IBW).

A. 108.15
B. 96.43
C. 100
D. 87.7

40. The indication for Edward’s degree of malnutrition is __________.

A. 3rd Degree
B. No malnutrition
C. 2nd Degree
D. 1st Degree

Situation – Field Health Services and Information System (FHSIS) provides a summary
of data on health service delivery and selected programs from the barangay level up to
the national level. As a nurse, you should know the process on how this information to
processed and consolidated.

41. All of the following are objectives of FHSIS, EXCEPT:

A. Ensuring that data reported are useful and accurate and are disseminated in a timely
and easy to use fashion.
B. Minimizing recording and reporting nurden allowing more time for patient care and
promotive activities.
C. Providing a standardized, facility level database which can be accessed for more in-
depth studies.
D. Completing the clinical picture of chronic disease and describe their natural history.

42. The fundamental block or foundation of the FHSIS is the _________.

A. reporting forms
B. target client list
C. output record
D. Family treatment record

43. The primary advantage of having a target client list is it _________.

A. lets nurses save time and effort in monitoring treatment and services to beneficiaries
B. facilitates easier reporting for the nurses
C. helps nurses monitor service rendered to clients in general
D. facilitates monitoring and supervision of services

44. The nurse uses the FHSIS record system INCORRECTLY when she __________.

A. records a child who has frequent diarrhea in TCL: Under Five


B. refers to other sources for completing monthly and quarterly
C. reports goes to the individual or FTR for entry confirmation in the Tally/Report
Summary
D. records diarrhea in the Tally sheet/Report from with a code FHSIS/M-1

45. In assessing a patient, the __________ is used by the nurse to record his/her address,
full name, age, symptoms, and diagnosis.

A. target client list


B. output record
C. individual treatment record
D. reporting form

Situation – Nurse Polistico is assigned under the Nurse Deployment Program in a rural
area in Northern Luzon where endemic malaria occurs.

46. During his assessment of a client, which among the following signs and symptoms of
malaria need referral to a secondary or tertiary facility?

A. Fever and chills


B. Sweating and headache
C. Weakness and Tachycardia
D. Icterus and shock

47. Laboratory confirmation of malaria is done on a blood film. What does Nurse
Polistico expect to see in the film?

A. Antibodies
B. Malarial parasites
C. Antigen
D. White blood cells

48. The mode of transmission of malaria is through the bite of an infected female
mosquito called __________.

A. Aedes Aegypti
B. Aedes poecillus
C. Anopheles
D. Culex

49. Which of the following is NOT an anti-malarial drug?

A. Tetracycline
B. Sulfadoxine
C. Amoxicillin
D. Quinidine

50. As a PREVENTIVE measure for malaria, Nurse Polistico educates people living in
malarial endemic areas on which of the following?

1. Avoid going out between 12nn to 3pm


2. Take chloroquine tablets once a day
3. Apply insect repellant on house walls
4. Use long sleeved shirts when going out at night
5. Plant neem tree in their backyards
6. Clear hanging tree branches near rivers

A. 1, 3, 5, 6
B. 2, 3, 4, 5
C. 1, 2, 3, 4
D. 3, 4, 5, 6

Situation – Occurrence of conflicts is a natural part of the workplace. Conflict


management is the ability to be able to identify and handle conflicts sensibly, fairly, and
efficiently.

51. Public health nurse Lily has become very disagreeable among her team members.
The nurses know that she has requested to be transferred to another Rural Health Unit.
The nurses decided to ignore the situation. The illustrates which method of conflict
resolution?

A. Collaboration
B. Avoidance
C. Accommodation
D. Passive-aggressiveness

52. Which actions are MOST important in preventing conflict to occur?

1. Allocate resources fairly


2. Keep problems confidential
3. Address staff fears
4. Clearly state expectations
5. Keep lines between roles flexible.

A. 2, 3, 5
B. 1, 3, 4
C. 2, 4, 5
D. 1, 2, 3

53. A health care worker approaches the public health supervisor with a complaint
about the RHU Nurse and said “She doesn’t do same amount of work as the other
health professionals in the Health Unit”. After investigating about the complaint, the
Nurse Supervisor believes the health care worker does not understand the role of the
RHU nurse. What kind of conflict has this situation cause the health worker?

A. Interprofessional conflict
B. Individual conflict
C. Intraprofessional conflict
D. Organizational conflict

54. Which of the following actions by the public health nurse indicates the use of
affective conflict management skill?

A. Defends judgement of others


B. Negotiates with others
C. Asserts own position
D. Avoid competition

55. The public health midwife was very upset regarding a reprimand issue by her
supervisor. The former confronts her supervisor in the hallway. How should the
supervisor respond?

A. Tell the midwife to calm down and act professionally


B. Respond to the midwife in the hallway
C. Ignore the situation and forget about it
D. Invite the midwife to come to the supervisor’s office
Situation – You will start working as a fiend nurse at the Philippine Red Cross. You are
reviewing RA 7719 or the Blood Services Act of 1994. This law ensures a safe and
accessible blood supply by promoting voluntary blood donation.

56. The following are characteristics of donor donating blood.

1. Be in good health condition


2. Weight at least 110lbs
3. Be 17 years of age and older
4. Carry a valid identification card
5. Be at least 5 feet tall
6. Never had and accident

To ensure the safety if blood supply, donors must meet which of the following BEFORE
donating?

A. 1, 3, 4, 5
B. 1, 4, 5, 6
C. 1, 2, 5, 6
D. 1, 2, 3, 4

57. You are aware that a person is STILL ELIGIBLE to donate blood in which of the
following conditions?

A. Taking antibiotics or maintenance medication


B. Pregnant and recent childbirth
C. Having high or low blood cholesterol
D. Having used self-injected, non-prescribed drugs

58. You keep a list of “Walking Blood Donors”. You mobilize the community to register
in this list with their blood types. Some instructions for blood donors are the following?

1. Register at any Blood Center Unit in a government hospital.


2. Go to any hospital or clinic and register as potential blood donor.
3. Your complete health history will be taken.
4. You will be given honorarium upon registration.
5. A physical examination and your vital signs will be taken.
6. A blood test will be taken to determine your blood type.

Which of these will you include in your instructions?

A. 2, 3, 4, 5
B. 1, 3, 5, 6
C. 1, 4, 5, 6
D. 2, 4, 5, 6

59. After donating blood, you advise the donor to do the following, EXCEPT to
__________.

A. enhances production of new blood cells


B. skip the next meal to allow the body system to rest.
C. leave the adhesive dressing from 3-6 hours.
D. avoid carrying or lifting heavy objects using the donating arm.

60. Which of the following statements is NOT true about the benefits of blood donation?
It __________.

A. Enhance production of new blood cells


B. Burns calories
C. reduces cancer risk
D. is a good prevention against infection

Situation – Health Centers regularly conduct “Operation Timbang” to assess the


nutritional status of children 0-5 years old.
61. The nutritional status of children is determined by weight by age. Weight is an
example of what level of measurement?

A. Ordinal
B. Ratio
C. Interval
D. Nominal

62. While age is of what level of measurement?

A. Ratio
B. Interval
C. Ordinal
D. Nominal

63. The sex of the children included in his study will be classified as male and female.
This measurement is:

A. Interval
B. Ordinal
C. Ratio
D. Nominal

64. The educational level of the parents is also recorded with the possible choices given
as:

- Elementary level
- Elementary graduate
- High school level
- High school graduate

The above set of choices is an example of what level of measurement?

A. Ratio
B. Interval
C. Nominal
D. Ordinal

65. The mothers of children are also asked particularly on the child’s health history.
One of the questions, “Did your child experience episodes of diarrhea within the last 6
months?”. If yes, skip item no 4 and item no. 5. This question item is referred to as:

A. Funnel question
B. Follow-up question
C. Contingency question
D. Filter question

Situational – Public Health Nurse have important roles of ensuring the well-being of the
family. They themselves should be empowered as they assist family clients is enabling
their capacities to maintain and improve their health status.

66. The nurse explains that families pass through predictive stages. This statement is
based on _________ models.

A. System models
B. Focal
C. Structural
D. Developmental

67. Which of the following would BEST indicate a maturational crisis?

A. Illness
B. Death
C. Marriage
D. Unemployment

68. Providing crisis intervention is an important part of _________.


A. primary prevention
B. health promotion
C. secondary prevention
D. Tertiary prevention

69. The nurse observes that the family is exhibiting strength in the face of hardships
they are facing. The best description of the strength the nurse is observing is __________.

A. power
B. resilience
C. projection
D. Faith

70. The nurse is educating the family regarding general needs for adequate nutrition,
rest, and physical activity. Which level of prevention does this describe?

A. Tertiary prevention
B. Secondary prevention
C. Health maintenance
D. Primary prevention

Situational – Public health nurses use their nursing skills in the application of public
health functions and provision of social assistance to their clients.

71. Which of the following statement BEST describes Public Health Nursing?

A. Services are the best implement only in far flung and hard-to-reach areas
B. Services are rendered free of charge to people in the catchment area
C. It involves home care to sick people who cannot be confined in the hospital
D. It puts emphasis on health promotive and preventive services

72. The current emphasis for public health practice is _________.

A. Improving environmental sanitation


B. controlling epidemic diseases
C. preventing communicable disease
D. advocating for social justice

73. The public health nurse must participate in the essential services that are relevant
and accessible to the family and the community. These include:

A. Working in enforcing laws to regulate health and ensure safety


B. Diagnosing and investigating health problems of the country
C. Informing, educating, and empowering the people about health
D. Monitoring health status and completing a community assessment

74. Collecting data and monitoring the health status of the population defines which of
the core public health function?

A. Quality assurance
B. Assessment
C. Policy development
D. Health promotion

75. Public health services are provided by government facilities. What is the most
appropriate response of a public health nurse to the statement: the government should
give these health services free of charge to people?

A. “Yes and for that reason, we should choose our government officials wisely during
elections”
B. “Yes i agree. But right now. The government does not have enough resources to do
that.”
C. “That is an ideal situation that our government should aspire to attain in the near
future”
D. “Many of the health services are given free, but we people pay for them just the
same through our taxes”
Situation – The ability to communicate is a very important skill for every nurse to have.
The nurse must do so in an effective, caring and professional manner,
especially when communicating with patients and their families.

76. A nurse wants to present information about influenza immunization to the older
adults in the community. Which type of communication should the nurse use?

A. Interpersonal
B. Public
C. Small group
D. Intrapersonal

77. A nurse is standing beside the patient’s bed. Nurse: How are you doing? Patient: I
don’t feel good. Which of the following communication ELEMENTS us categorized as
feedback?

A. “How are you doing?”


B. “I don’t feel good”
C. Nurse is standing
D. Patient in bed

78. A smiling patient angrily states, “ I will not cough and deep breathe”. How will the
nurse interpret this finding? The patient’s __________.

A. signified meaning is wrong


B. vocabulary is poor
C. affect is inappropriate
D. personal space was violated

79. BEFORE meeting the patient, a nurse talks to other caregivers about patient. Which
phase of the helping relationship is the nurses in with this patient?

A. Termination
B. Working
C. Orientation
D. Preinteraction

80. During the INITIAL home visit, the nurse lets the patient know that the visits are
expected to end in about a month. Which phase of the helping relationship is the nurse
in with this patient?

A. Pre-interaction
B. Working
C. Temination
D. Orientation

Situation – As a Public Health Nurse in your local community. It is one of your main
responsibilities to educate the community about good health practices.

81. During community forum. You discussed about herbal medicine. You explained to
the participants that “Botika sa Paso” is done by __________.

A. cultivating herbal plants in vacant or unused lots


B. growing herbal plants in pots
C. planting herbs along the barangay streets
D. experimenting with herbal decoration

82. You also discussed about having a “Healthy Market”which means that the
community should _________.

A. sell fresh and healthy foods only


B. organize recycling teams
C. collect garbage and recycle them
D. dispose waste and garbage separately
83. As part of your topic, your shared about the strategy that should be initiated by
students and teachers for a “Healthy School”. What is this strategy about?

A. Implementing Environmental Sanitation Activities


B. CPR demonstration
C. Selection of better school doctors and school nurses
D. Scrap booking using recycled materials

84. One of the participants asked you about what herbal plants to use when treating
diarrhea. You answer that young leaves of ________ can be used as a home remedy.

A. Tsaang gubat
B. Sambong
C. Lagundi
D. Bayabas

85. One of the new mothers attending the discussion wants to know about vaccination.
One of the key information you share is that __________.

A. diarrhea and malnutrition are contraindicated


B. if the vaccination schedule is interrupted, it is necessary to restart
C. it is safe to vaccine a sick child who is suffering form minor illness
D. minor infections with low fever (below 38.5 Celsius) is contraindicated

Situational – Public Health Nurse Maria is in charge of protecting the vaccine supply in
the Rural Health Unit. In performing this responsibility, she is guided by the standards
and guidelines in “How to Protect the Vaccine”.

86. The type of refrigerator/freezer utilized to store vaccine in a health center should
be:

A. Standard chest-type freezer


B. Standard refrigerator with separate freezer door and seal
C. Dormitory type refrigerator and separate dormitory type freezer
D. Dormitory type refrigerator with small hanging freezer inside

87. Which of the following vaccine go in the freezer?

A. Hepatitis B
B. Varicella or Chickenpox
C. Tetanus
D. Diptheria

88. The temperature in the refrigerator and freezer should be checked:

A. Twice a month
B. Twice a day
C. Once a day
D. Once a week

89. Vaccines should NEVER be stored in which part of the refrigerator?

A. Level right-hand corner


B. Level left-hand compartment
C. Front
D. DOOR

90. Vaccines can be mixed in a single syringe when:

A. there is need to decrease the number of injections to be given


B. remaining vaccines need to be consumed during the day
C. giving all live or all inactivated vaccines
D. vaccines are licensed and labeled to be mixed

Situational – A 6-5 magnitude earthquake struck last April 13, 2019 in municipality X.
Mary Grace was a trained nurse in emergency and disaster preparedness.
91. As a primary healthcare giver, which of the following activities should Nurse Mary
Grace NOT involve in?

A. recovery of primary health care activities


B. water safety and quality
C. food safety and food security
D. search, rescue, and evacuation

92. One function of Mary Grace in emergency and disaster is to provide immediate care
EXCEPT:

A. victims identification and reporting


B. trauma care
C. To coordinate with military
D. first aid, triage, and transport of victims

93. A resident in Macasandig by the name Aldin Richard sustained injury. First Aid was
initiated by nurse Mary Grace. This is given __________.

A. during the arrival of an ambulance or doctor


B. during the arrival of an ambulatory doctor or other qualified personnel
C. before the arrival of an ambulance or doctor
D. during the arrival of an ambulance, doctor or other qualified personnel

94. Which of the following is NOT true regarding nurses responsibilities in disaster
preparedness?

A. Nurses will be the leader in the incident command structure set up at the disaster
site
B. Nurses have a personal and family plan as a part of their disaster preparedness plan
C. Nurses who are prepared for disasters will be better able to help themselves, their
families, and their communities in a disaster situation
D. Nurses have a responsibility to the public to be knowledgeable about disaster
preparedness response

95. Mary Grace consider the following factors in patients triage EXCEPT__________.

A. pulse and respiration


B. temperature
C. mental status
D. Ability to walk

Situation – A public health nurse is giving orientation to three beginning nurses who are
hired by the local government unit under the Nurse Deployment Program.

96. In the first session, the nurse is presented the organizational chart of the Rural
Health Unit. Which of the following aspects of organizational structure is illustrated in
the chart?

1. Type of work being done


2. Extend of coordination to be done
3. Levels of management
4. Line and staff relationship

A. 1 and 3
B. 2, 3, 4
C. 3 and 4
D. 1, 3, 4

97. The PHN wants to emphasize to the new nurses that it is important to observe
professional accountability which is taking responsibility for one’s action. She explains
that accountability can be done by:

1. asking assistance from the doctor


2. performing nursing tasks in a same manner
3. reporting and documenting assessment and interventions
4. evaluating client’s response to nursing care given
5. evaluating the care when there is complain by the client

A. 2, 4, 5
B. 1, 3, 5
C. 2, 3, 4
D. 1, 3, 4

98. Accountability also includes a commitment to continuing education to stay current


and knowledgeable. According to the Guidelines Implementing Continuing Professional
Education or Nurses (CPE), from which of the following can a nurse earn credit units?

1. Participation in seminars
2. Engagement in research projects
3. As peer reviewer
4. Innovation

A. 1, 3, 4
B. 2 and 3 only
C. 1, 2, 3, 4
D. 1 and 2 only

99. The PRIMARY purpose for requiring continuing education is to ___________.

A. quality for work abroad


B. improve nursing competency after registration
C. comply with the Board of Nursing requirements
D. ensure legal practice

100. To better address emerging public health issue, a public health nurse enrolls in a
course in:

A. Leadership
B. Ethics
C. Communication
D. Research
NURSING PRACTICE II

Situation - Cel is a beginning nurse. She still has to be mentored when it comes to
charting.

1. Cel must know that in narrative charting, documentation of client care should be
____. Choose the BEST answer.

A. Descriptive
B. Formatted
C. Extensive
D. Chronologic

2. Which statement is not TRUE about medical record that Cel must understand?
Charts ______.

A. Can be accessed by researcher.


B. Contents must be kept confidential.
C. Can be given out only with client’s written consent.
D. Can be borrowed by any nursing student.

3. The following are the methods of charting, EXCEPT:

A. Focus
B. Narrative
C. POMR
D. Sequential

4. In the PIE method of charting, the meaning of P is _______.

A. Discharging
B. Teaching
C. Admission
D. Planning

5. In charting neurological assessment, Cel must understand that the following should
be indicated EXCEPT:

A. Skin elasticity
B. Facial symmetry
C. movement of extremities
D. level of consciousness

Situation - Baby Sharon, a newly delivered baby girl was delivered normally at the
maternity clinic. Nurse Juvy is performing her initial routine assessment.

6. Nurse Juvy’s assessment reveals the following: Heart Rate is 110 beats per minute,
has a vigorous cry, moves actively and with good flexion, normal skin color and bluish
extremities. What would be the APGAR Score of baby Sharon?

A. 7 points
B. 10 points
C. 5 points
D. 9 points

7. Baby Sharon was placed on photo therapy. What precaution should Nurse Juvy
observe?

A. Put sunglasses on the newborn to protect his/her eyes


B. Be certain that the newborn’s intake is adequate
C. Assess the newborn for symptoms of headache
D. Keep the newborn wrapped to prevent sunburn.

8. Physiologic jaundice among newborn babies usually occur on, which of the following
it occurs ______.

A. Within 24 hours from birth


B. 7 days after birth
C. Upon birth
D. Between the 2nd and the 3rd day after birth

9. If transient discoloration of Baby haron’s skin is noted while under phototherapy.


What is this phenomenon called?

A. Cyanosis
B. Jaundice
C. Hyperbilirubinemia
D. Bronze baby syndrome

10. If Baby Sharon develops dehydration, what is the FIRST sign to look or by Nurse
Juvy?

A. Oliguria
B. Sunken Fontanels
C. Soft and depressed eyeballs
D. Non-elastic skin/poor skin turgor on thighs and abdomen

Situation - A nurse assigned in a neonatal intensive care unit receives a telephone call
from the delivery room and is told that a newborn with spina bifida (myelomeningocelle
type) will be transported to the unit immediately.

11. Which of the following definitions MOST accurately describes meningomyelocele? It


is a _______.

A. Spinal cord tumor containing nerve roots


B. Complete exposure of the spinal cord and meninges
C. Herniation of spinal cord, cerebro-spinal fluid and meninges into a sac
D. Sac formation containing meninges and spinal fluid

12. The NICU nurse prepares for the arrival of the newborn. Which of the following
PRIORITY items. Should it be placed at the newborn’s bedside? A ______.

A. Specific gravity urinometer


B. Rectal thermometer
C. Blood pressure cuff
D. Bottle of sterile normal saline

13. For this patient who is undergo surgery (closure of the sac), what would be the
PRIORITY nursing diagnosis?
It is risk for ______.

A. Activity intolerance
B. infection
C. Respiration
D. Altered growth & development

14. Which of the following is the PRIMARY reason for surgical repair of
myelomeningocele? To _______.
A. Decrease risk of infection
B. Correct the neurologic defect
C. Prevent seizure disorders
D. Prevent hydrocephalus

15. What would be the BEST response of the nurse when a mother with neural tube
defect asks what she can do to decrease the chances of having another baby with the
same defect?

A. “Folic acid should be taken before and after conception.”


B. “Multivitamins supplements are recommended during pregnancy.”
C. “A well-balanced diet promotes normal fetal development.”
D. “Increased dietary iron improves the health of mother and fetus.”

Situation - Patient Edraline, 29 years old visits the OB clinic accompanied by her
husband Victor’s pregnancy test. The nurse in the clinic took her history which says
that Edraline missed her periods for three (3) weeks.

16. Which of the following is the PRIMARY purpose of a pregnancy test?

A. It allows for counseling on nutrition.


B. It may help in a decision to stop working at home.
C. It enables the husband to follow the desires of the wife.
D. It allows for early initiation of care

17. Human Chorionic Gonadotropin (HCG), the biologic marker on which pregnancy
tests are based can be detected in the BLOOD as early as which number of DAYS after
the last menstrual period?

A. 15
B. 20
C. 10
D. 5

18. What is the BEST urine sample to be used for an accurate result of a pregnancy test
done at home?

A. First-voided evening urine


B. First-voided morning urine
C. Mid-afternoon urine sample
D. Random urine sample

19. Which of the following is NOT included among the factors that should be considered
by the nurse of the pregnant woman in interpreting results of pregnancy test?

A. Last menstrual Period


B. Previous pregnancy
C. Type of pregnancy Test
D. Menstrual cycle length

20. Which of the following medications may produce a false-negative pregnancy result?

A. Anti-hypertensives
B. Anti-convulsants
C. Diuretics
D. Tranquilizers

Situation - A vehicular accident occured recently in Barrio Kapitan. About were injured.
One of them is Almira, G3P4, 6 months pregnant, her son Jay, 4 years old, the driver, the
conductor and the rest were students. Some had minor injuries, Jay had minor
lacerations on his arm and right leg. One student had Colle’s fracture, the conductor
had a broken leg while the driver had lacerations on his head. Almira, on the other
hand, complained of abdominal cramps and vaginal spotting. A rescue team from a
nearby hospital arrived immediately on the scene.
21. Who among the passengers will take PRIORITY for the rescue team to transport to
the hospital?

A. Student with Colle’s fracture


B. Jay with laceration
C. Pregnant woman
D. Conductor with broken leg

22. In admitting the injured patients, which of the following should be the FIRST that
should be done by the emergency team? They should assess the patient’s ______.

A. Breathing
B. Airway
C. Circulation
D. vital signs

23. In assessing rooms for the injured patients, the nurse should coordinate with the
Administration. Which of the following is the CORRECT room assignment?

A. Insurance company should be followed according to premiums paid by the transport


operator.
B. Mother should be separated from each other for case classification.
C. Mother and Child should be together in one room
D. Transport operator should designate the rooms according to social status of patients.

24. In as much as Almira complained of vaginal spotting and abdominal cramps. Which
among the following will the nurse anticipate as the MOST likely diagnosis of the
physician after a vaginal examination?

A. Eclampsia
B. Placenta Previa
C. Threatened Abortion
D. Abruptio Placenta

25. One GOOD nursing intervention of the nurse for Almira would be to do. Which of
the following?

A. Advise her to eat her meals.


B. Put her on absolute bedrest.
C. Take the vital signs q4 hours.
D. Advise her to take care of Jay.

Situation - Nursing Service Management should endeavor to update nurses for the
improvement of quality and safe nursing care, especially with the passage of the
universal Health Law.

26. How can nurse contribute to the improvement of Maternal and Child Health (MCH)
in the Philippines? One way is by knowing the 8 Millenium Development Goals. Which
of the TWO of the 8 Goals are VERY specific to MCH?

A. Reduce child mortality and improve maternal health


B. Eradicate extreme poverty and hunger and achieve universal primary education
C. Promote gender equality and empower women and global partnership for health
D. Combat HIV/AIDS and ensure environmental sustainability

27. The Universal Health Law focuses on the population. If a pregnant woman has been
found and diagnosed to have pre-eclampsia, the focus of health care is on the _______.

A. Population Group
B. Individual
C. Community
D. Family

28. Which among the following is TRUE about mother and child health in the
Philippines (among ASEAN) that can be addressed by the Universal Health Law?
1. Fastest decline in maternal mortality rate
2. Number one highest in infant mortality rate
3. Fastest rising HIV/AIDS
4. Highest TB burden

A. 2, 3, 4
B. 1, 2, 3
C. 1, 2
D. 3, 4

29. In the Universal Health Law, which of the following are the government agencies
that are considered as “key players” in its implementation?

A. Family, DOH and Philhealth


B. DOH, LGU, and Philippine Health Insurance, Inc. (Philhealth)
C. Community, DOH, LGU
D. Individual, DOH and LGU

30. What is one of the MAJOR challenges that a beginning nurses may encounter in her
service delivery in the hospitals. In terms of an INDIVIDUAL client or patient?

A. Dysfunctional facilities
B. Poor health seeking behavior
C. Social determinants of health
D. Stock out of essential commodities

Situation - Everyday, nurses make ethical decision in their nursing practice. This
decision may involve patient care, action related to co-workers or nurse-doctor
relationship. Nurse Lydia is assigned in the Obstetrics Ward.

31. Which of the following will NOT guide Nurse Lydia in her ethical decision?

A. There are automatic solutions to all ethical conflicts.


B. Nurse is not legally responsible for obtaining a patient informed consent.
C. Legally Nurse Lydia is responsible for using her knowledge and skill to protect her
patient.
D. Nurse Lydia can act as a patient advocate.

32. Nurse Lydia considers consulting the Ethics Committee. Which of the following
considerations given by the Ethics Committee is applicable?

A. The family members disagree about what should be done by the nurse.
B. The health care team disagree among themselves about the treatment.
C. The doctor disagree with the patient & his family regarding treatment
D. All of these

33. Patients must be notified about their rights to privacy and how their health
information will be used and shared by the health team. He also has the right to decide
to whom will he allow the information be shared. This right is, which of the following?

A. Beneficence
B. Justice
C. Informed Consent
D. Confidentiality

34. Ethics Committee provides as forum for the patient, his family and health care team
to resolve a conflict. In this committee, who among the following cannot be a member?

A. Nurse
B. Lay person/clergy
C. lawyers
D. Doctor

35. Who among the health team can BEST give the disclosure to patient about informed
decision regarding her future medical care?
A. Nurse
B. Significant others
C. Doctor
D. Midwife

Situation - Nadine, three years old, was admitted to the Pediatric Ward due to frequent
coughing, presence of wheezes when breathing out and chest congestion. Upon
physical examination, Nurse Alma noted some bruises on the back and lower
extremities of Nadine. When the mother was asked of the noted bruises, the mother
said it is due to Nadine’s being hyperactive and playful. Mother of Nadine said that the
child is bed-wetting. Nurse Alma also observes that Nadine also shows sign of anxiety.

36. The toddler years are a time of great cognitive, emotional and social development.
The toddler is a child _____ months old.

A. 6 to 12
B. 9 to 36
C. 36 to 48
D. 12 to 36

37. Upon seeing warning signs of child abuse, the BEST nursing action that Nurse Alma
should make is to report the noted observation to______.

A. Dept. of Social Welfare Development (DSWD)


B. Headnurse
C. Philippines National Police
D. Attending Physician

38. Nurse Alma recalls that there are four kinds of child abuse. Which of the following
below are considered as child abuse? Select all that apply.

1. Physical
2. Emotional
3. Sexual
4. Neglect

A. 1 & 3
B. 1, 2 & 3
C. 1, 2, 3 & 4
D. 1 & 4

39. When there is a failure to supervise a child adequately, especially in children


younger than 12, the kind of child abuse is, which of the following?

A. Emotional
B. Sexual
C. Neglect
D. Physical

40. Since Nadine is four years old, Nurse Alma needs to determine the routines and
rituals concerning, which of the following, EXCEPT_____.

A. Feeding
B. Toilet training
C. Sleep Pattern
D. Favorite toy(s)

Situation - Nurse Angie is conducting a mother’s class at Barangay Riles Rural Health
Unit. One of the participants asked about contraceptive pills.

41. As part of the teaching plan, Nurse Angie teaches that oral contraceptives contain
estrogen. Which of the following is the ACTION of estrogen? It inhibits the ______.

A. GnRH thereby inhibiting FSH and LH production


B. Luteinizing hormone (LH) thereby inhibiting ovulation
C. Testosterone productions
D. Follicle stimulating hormone (FSH) thereby inhibiting maturation of ovum

42. Nurse Angie added that oral contraceptives also contain progesterone. Which of the
following is the action of progesterone in contraception? It inhibits ______.

A. Testosterone production
B. GnRH thereby inhibiting FSH and LH production
C. Luteinizing hormone (LH) thereby inhibiting ovulation
D. Follicle stimulating hormone (FSH) thereby inhibiting maturation of ovum

43. Nurse Angie included that contraceptives that have estrogen – like and/or
progesterone - like compounds are prepared in a variety of forms. These are the
following ____.

1. Oral contraception
2. Diaphragms
3. Cervical caps
4. Foam spermicides
5. Norplant
6. Intrauterine device

A. 2, 3, 5
B. 3, 5, 6
C. 1, 4, 6
D. 1, 5, 6

44. A biphasic anti-ovulatory medication of combined progestin and estrogen is


prescribed for a female patient. What should the nurse include when teaching about
this oral contraceptive?

A. Restrict sexual activity temporarily


B. Report any irregular vaginal bleeding
C. Increase the intake of calcium.
D. Have bi-monthly Pap Smears

45. Which danger sign would warrant stoppage of pills?

A. Clear vision
B. Decrease blood pressure
C. Simple colds
D. Elevation of blood pressure

Situation - A young mother brings her 4 year-old. Richard to a Pediatric Clinic where
you are assigned.

46. The young mother wanted to know about the motor development APPROPRIATE in
a preschooler. Which of the following statements is NOT true?

A. “He can tie shoe lace.”


B. “He can alternate feet when climbing.”
C. “He has not developed good posture.”
D. “He can hop two or more times.”

47. Healthy physical development is dependent upon nutrition, brain development,


muscle and bone. Which of the following is NOT APPROPRIATE for physical
development of a pre-schooler?

A. Sleep 6 to 8 hours of sleep each day.


B. Assist in brushing and flossing teeth
C. Gain 5 pounds per year
D. Eruption of the primary teeth

48. Her son starts asking you many questions about the things inside the Pediatric
Clinic. You should inform the mother of which of the following statements?

A. Expected in a preschooler as they tend to ask many questions during this age.
B. Expected in toddlers only so he may have a delayed development
C. Not normal in a preschooler so the mother say need to consult a child psychiatrist.
D. Impolite so the mother should discipline her son so stay quiet in public places.

49. You informed the mother about the normal psychosocial; development of
preschoolers. She correctly understands your health teaching if she verbalizes that
_______.

A. He may have temper tantrums resulting from his frustration in wanting to do


everything for himself.
B. He continues to react to separation from his parents
C. Her son is more active with his parents and tends to be a bit selfish with his toys
D. He may tend to exaggerate, boast and tattle on others.

50. Before finishing the check-up, which of the following principles is included among
the principles of guidance in handling Richard?

A. Controlling temper tantrums


B. Basing her expectations within the child’s limitations
C. Acceptance of masturbation as a normal phenomenon to be discourage in public
D. Reinforcing the correct use of language.

Situation - Patient Haydee, 31 years old, G2 P1 and is in her 9th month of pregnancy.
First, she calls the hospital to receive validation that it is alright for her to come for
evaluation of admission.

51. During the first contact of the patient with the nurse, the letter should demonstrate
the following behavior, with the EXCEPTION of _____.

A. Caring
B. Comforting
C. encouraging
D. Compelling

52. Patient Haydee asks how she could distinguish between true and false labor? Which
is NOT INCLUDED among the factors in which the Nurse should base her answer from?

A. Contractions
B. Cervix by vaginal examinations
C. Vital signs
D. Engagement of fetus

53. Patient Haydee comes to the perinatal unit of Hospital Dee. Nurse Arcee does a
thorough SCREENING assessment. Which is the LEAST screening assessment to be
used by the nurse?

A. Physical examination
B. Interview
C. Radiologic procedure
D. Laboratory review

54. During admission, the nurse needs to take the patient’s obstetrical data. Which of
the following is the MOST important?

A. Laboratory Results
B. Prenatal check up records
C. Previous pregnancy experience
D. Diagnostic test results

55. Every pregnant woman preparing for labor and delivery has a birth plan. What is
the PRIMARY objective of a birth plan? It describes ______.

A. The repertoire of comfort and relaxation measure


B. Options on her wishes and preferences about her labor, delivery and puerperium
C. The cultural and religious requirement related to care of mother, newborn
D. Her psychosocial data that is necessary in her hospitalization
Situation - Through health practitioners such as the nurse, provision of quality health
care and patient safety is made possible. Hence, there is a Nurse of a 100-bed capacity
private hospital. Lately, her Head Nurses have been complaining of short staffing
especially in the Pediatric Ward. The ideal staffing in her ward cannot be attained as
nurses frequently leave for abroad for a greener pasture despite having signed a
contract.

56. When the staff nurses in the Pediatric Ward work more than they can handle, thus
losing their enthusiasm in their work. Which of the following consequences is referred
to?

A. Lower patient care


B. Medical error
C. Low quality patient care
D. Burn-out

57. Pediatric Nurses need to organize their workload to facilitate patient’s safety. Which
of the following indicators would validate that they have organized their workload?
Select all that apply.

1. Determination of resources needed to deliver patient care.


2. Accomplishment of tasks on time.
3. Identification of activities to be done.
4. Prioritization of plans in accomplishing activities to be done.

A. 1, 2, 3, 4
B. 1, 2 & 3
C. 2, 3 & 4
D. 1 & 2

58. Despite the short staffing, Nurse Cirila also has the responsibility to maintain a safe
environment for her Pediatric patients. The following are the indicators that Nurse
Cirila has achieved this goal, EXCEPT, ______.

A. Proper disposal of waste according to color coding


B. Adherence to policies, protocols and procedures in the prevention and control of
infection prevention.
C. Assessing the competencies of a staff before delegating a task
D. Observance of protocols in cases of earthquakes, fire and other emergencies

59. Chief Nurse Cirila has identified common reasons for the fast turn-over of staff
Nurses not only in the hospital she works in but in other hospitals all over the country.
Which of the following factors that have contributed to the fast turn-over should be
addressed immediately?

1. Low salary
2. Working condition
3. Work Overload
4. Continuing professional Development

A. 1 & 3
B. 1 & 3
C. 1, 2 & 3
D. 1, 2, 3 & 4

60. Chief Nurse Cirila formulates strategies to address to short staffing in her hospital.
Which of the following strategies is the BEST?

A. Continuing professional Development such as Post-graduate study


B. Entry salary of Nurses to be at par with that of government nurses.
C. Research engagement
D. Evaluation and recognition for good performance thru reward system

Situation - Tricia is a newly Registered Nurse who is assigned to conduct a lecture on


fertility awareness among young women.
61. Tricia discussed the signs and symptoms of ovulation. Which of the following is NOT
included?

A. Rise in Basal Temperature


B. Weight gain
C. Breast tenderness
D. Increased quantity and PH of the cervical secretions

62. One of the participants asks “when does ovulation take place?” Tricia’s answer will
MOST likely be _____.

A. On the 14th day before the next menstruation


B. On the 4th day of her cycle
C. As soon as menstruation stops
D. At the time her basal body temperature drops steadily.

63. Melissa, one of the participants asks how many days is her menstrual cycle if she
had her previous menstrual flow last March 26-30 and her last menstrual period last
April 24th. Most likely, the nurse’s response will be ____days.

A. 30
B. 28
C. 29
D. 31

64. The function of estrogen hormone is, which of the following?

A. Promotes lactation
B. Promotes female secondary sex characteristics
C. Inhibits uterine contractility
D. Elevates body temperature during ovulation

65. The space of the menstrual cycle that promotes the uterine lining to grow and
thicken to an 8-10-fold rate is, which of the following?

A. Menstruation
B. Proliferative
C. Ischemia
D. Secretory

Situation - Aira, student, 18 years old, primigravida, delivered to a baby boy forty hours
ago. Transition from being single to being a mother entails strengths and courage. The
response to change maybe different for every person.

66. The patient has been seen crying and irritable. As her nurse, you know that Aira is
experiencing “baby blues”. Which is the BEST description of her condition? It is a
condition in which the patient experiences some feelings of _____.

A. Excitement
B. Euphoria
C. Sadness
D. Anxiety

67. Which of the following is TRUE about baby blues? It is ______.

A. A condition that begins 6 to 12 months postpartum


B. A serious condition that would warrant antidepressant therapy
C. Related to hormonal changes
D. Related to childhood poverty

68. In planning to help Aira, the nurse has explained her on the cause of baby blues,
which ONE is these

A. Total increase in estrogen and progesterone.


B. Status quo in estrogen and progesterone.
C. Total decrease in estrogen and progesterone
D. Increase or decrease on the levels of estrogen and progesterone.

69. As Nurse, you should know the manifestation that Aira can present anytime. These
are the following:

1. Crying
2. Anorexia
3. Feeling of inadequacy
4. Isolation
5. Disturbed sleep
6. Mood swings

A. 2, 3, 4, 5
B. 1, 2, 3, 5, 6
C. 3, 4, 5, 6
D. 2, 3, 4, 5, 6

70. Which of the following nursing measures can help Aira in her condition?

A. Counselling
B. Support
C. Psychotherapy
D. Mild anti-depressant

Situation - Maria Theresa, a Newly Registered Nurse is assigned in the Pediatric Unit.
She is aware of the legal implications of giving nursing care to patients.

71. A 10-year-old boy is refusing to take his medication. Maria Theresa threatened the
boy that he will be restrained. Which of the following will Maria Theresa be accused of
committing?

A. Assault
B. Battery
C. Malpractice
D. Negligence

72. A child fell off the bed and injured himself. Which of the following can the nurse be
sued for?

A. Battery
B. Malpractice
C. Negligence
D. Assault

73. Maria Theresa recalled that the criteria for negligence are the following, EXCEPT
_______.

A. There exists a duty


B. Breach of duty
C. Exceeding a limit of one’s duty
D. Injury or damage results

74. The patient sued the nurse for negligence. The patient's lawyer must prove the
following elements for professional negligence are satisfied. Which of the following
elements must the lawyer prove for professional negligence?

A. Duty, breach of duty and damages.


B. Breach of Duty, damage and causation
C. Duty, breach of duty, damages and causation
D. Duty, damages and causation

75. While the nurse is preparing the articles needed for the procedure, she overheard
the medical technologist telling the patient in a derogatory manner that the nurse
“doesn’t know anything about the procedure”. What can medtech be sued for?
A. Slander
B. Battery
C. Assault
D. Libel

Situation - JP, a nurse assigned in Barangay Obrero called for mother’s class. There
were a group of primiparas and several others bringing along their children.

76. While discussing prenatal care with a group of pregnant women, one of the women
asks the nurse, “What should I eat to ensure normal growth of my baby?” The nurse’s
response depends on an understanding of nutritional needs during pregnancy. Which of
the following statements is INCORRECT about nutrition in pregnancy?

A. Weight is the best way to measure the woman’s nutritional intake.


B. Adequate calcium can be provided by drinking 4 glasses of milk in a day.
C. There is a need to add 300 calories to the non-pregnant caloric intake of 1500
kcal/day.
D. Nutritional counselling should start with an assessment of the woman’s daily intake-
take a 24-day diet history.

77. You encouraged a first-time mother to talk to her child, a 9 month-old girl, who at
this stage should be _____.

A. Saying “dada”
B. Cooing when talked to
C. Obeying simple commands
D. Vocalizing single syllables

78. Another new mother is concerned about the developmental milestone of her son
who is 7 months old. Which of the following statements by the new mother indicates
that your health teaching was effective?

A. “He can stand up from sitting position.”


B. “My son can sit without support.”
C. “He can crawl.”
D. “He can drink from his training cup with minimal spilling.”

79. Another baby, Marites 6 month old, is brought in by her mother who is concerned
that her infant may have a delayed development. Upon assessment, you would expect
the infant to ____.

A. Be able to hold a place of a small snack with her finger and thumb
B. Reach out for her toy
C. Sit upright for a long period.
D. Hold her feeding bottle for a short while

80. A young mother asks you about the attachment behaviors she would expect her 4
month-old son to demonstrate. Which of the following is NOT included in your health
teaching?

A. The infant is friendly to strangers.


B. The infant follows his mother as she moves.
C. The infant greets his mother when she returns.
D. The infant recognizes and responds to his mother’s voice.

Situation - Nurse Hope is a beginning nurse assigned in the Pediatricians unit in a


government hospital. The unit has pediatric patients of different ages. Since she will be
dealing with patients of varied ages, she thought it best to review her notes on human
growth and development, specifically, Erikson’s psychosocial theory.

81. The BEST reason why Nurse Hope opted to review Erikson’s psychosocial theory is,
which of the following statements?

A. Failure to master these tasks leads to feelings of inadequacy.


B. Helps children grow into successful and contributing members of society.
C. Completion of tasks results in a sense of competence and a healthy personality.
D. We are motivated by the need to achieve competence in certain areas of our lives.

82. Nurse Hope immediately responds to any cry from her pediatric patients because it
is, which of the following reasons?

A. Lessen the noise overload in the Unit.


B. Attend to her patients who cannot communicate verbally
C. Check if the child is hungry or wet.
D. Be a powerful influence over that individuals interactions with others for the
remainder of his/her life

83. Nurse Hope is attending to a two-year-old Sheila, who is admitted due to chronic
bronchitis. Sheila sports long hair that extends up to her shoulder. As part of the
morning care, Nurse Hope decided to style Sheila’s hair into a ponytail. However,
Sheila vehemently resisted her hair being tied by a rubber band. Hope’s BEST thing to
do is, which of the following?

A. Deny Sheila’s preference


B. Allow Sheila’s preference
C. Explain that a ponytail would make Sheila more beautiful
D. Assert her authority

84. Carlito, 17 years old, is admitted due to influenza. He is admitted in a private room.
In one of Nurse Hope’s conversations with Carlito, the patient confessed that he is very
unhappy with the program he is taking up in college as this is not his choice but rather
the choice of his parents. In which of Erikson’s stages of development does this case
fall?

A. Autonomy versus shame/doubt


B. Trust versus Distrust
C. Integrity versus Despair
D. Identity versus Role Confusion

85. Myla, 7 years old, is confined in the Pediatrics Ward due to diarrhea. She is hooked
to an intravenous infusion. Sheila and family were taught to use the buzzer, should they
need something. Myla pressed the buzzer while her yaya was sleeping when her IV was
about to be consumed. The gesture of praising at this age is an example of which of the
following?

A. Generativity
B. Initiative
C. Industry
D. Autonomy

Situation - Pamela, 29 years old, has just given birth 3 hours ago. She asked Nurse
Eden to help her cope post-partially. One of the discussions was about the postpartum
vaginal discharge called lochia.

86. The nurse assessed Pamela’s perineum and noticed that the lochia discharge is
moderate in amount and red in color. What type of lochia is this? Lochia ______.

A. Fireum
B. Serosa
C. Rubra
D. Alba

87. When Pamela was helped for ambulation for the first time, she mentioned that she
had heavy lochial discharge. Which of the following assessment findings would BEST
help the nurse decide that the flow is within normal limits?

A. Her flow is over 500ml.


B. The color of the flow is red
C. Her uterus is soft to your touch.
D. The flow contains large clots.
88. Which of the following findings would make Nurse Eden think that Pamela is
developing a postpartum complication?

A. An absence of lochia
B. Lochia is the color of menstrual blood.
C. Red-colored lochia for the first 24 hours
D. Lochia appearing brown on the third day

89. What is the condition of Pamela if pinkish red or reddish brown-colored vaginal
discharge continues to occur after 3 to 4 weeks after birth with fever, pain and
abdominal tenderness?

A. Vaginitis
B. Cervicitis
C. Mastitis
D. Endometritis

90. It is difficult to judge the amount of lochial flow based only on observation of
perineal pads. Which among the following factors should be considered as IMPORTANT
indicator of complete and accurate measurement of lochia? This is the number of
perineal pads saturated and ______.

A. Time
B. Smell
C. consistency
D. Color

Situation - Nurse Rose is a staff nurse in the Maternity Ward of Zigzag Hospital. While
working as a staff nurse, she also pursues graduate studies for a Master’s Degree in
Nursing. She plans to conduct a study in her unit.

91. Nurse Rose understands that the main objective in conducting research in nursing
is to ______.

A. Develop the nursing profession


B. Improve nursing care
C. Ensure accountability for nursing practice
D. Document the cost effectiveness of nursing care

92. Nurse Rose decides to do a phenomenological study. What are the characteristics of
a phenomenological study?

A. Was initially developed by anthropologists to study the way a human being reacts
within or experiences a natural setting.
B. Enables the investigator to discover the theory from systematically obtained data.
C. Provides an in-depth description of the phenomenon of interest to the investigator.
D. Examines maternal experience as they are lived and understood as reality as human
beings.

93. Nurse Rose is aware that the phenomenological method of research is_____.

A. That is uses primary and secondary sources


B. A description of events from the past
C. The truth is a lived experience
D. That is involves field work

94. To collect data for her study, Nurse Rose interviewed six patient’s who had
experienced complications during pregnancy and after delivery. She interviews the
patient two or three times until the point of saturation has been reached. To what does
the term “saturation” in qualitative research refer to?

A. Sample size
B. Data repetition
C. Subject exhaustion
D. Researcher exhaustion
95. Nurse Rose drafts a title for her proposed study. Which of the following would be
the APPROPRIATE title for her study?

A. “Caring Behaviors of Nurses Toward Maternity patient’s with Complications.”


B. “Lived experiences of Maternity Patients with Complications.”
C. “Quality of life of Maternity Patients with Complications.”
D. “Competencies of Nurses in the Maternity Unit as Perceived by patients.”

Situation - Mark, 18 years old, is admitted due to Human Immunodeficiency Virus (HIV)
manifesting signs and symptoms of fever, headache, malaise, enlarged lymph nodes and
loss of appetite. Mark and family are so worried due to fear of being discriminated,
receiving judgmental attitudes from the hospital staff and disclosure of his admitting
diagnosis.

96. A part of Nurse Isabel’s orientation upon admission is presenting the Patient’s Bill
of Rights. When Nurse Isabel requests the parents to sign the admission form, which of
the following Patient’s Bill of rights is referred to?

A. Right to be informed of his rights and responsibilities as a patient.


B. Right to informed consent.
C. Right to information.
D. Right to privacy and confidentiality.

97. In signing the admission form, which of the following persons may sign the
document (in the order of priority)?

1. Either parent
2. Second degree relative
3. Brother or sister of legal age
4. Legal Guardian

A. 1, 2, 4 & 3
B. 1, 3, 2 & 4
C. 1, 3, 4 & 2
D. 1, 4, 3 & 2

98. To validate that Mark and family have understood HIV and the discharge plan made
by Nurse Isabel, to include the doctor’s order of medications to be taken at home,
which of the following statements is INCORRECT?

A. “HIV during adolescence are infected through sex.”


B. “HIV may also be contracted by mosquito bites, saliva, tears or sweat.”
C. “Medication adherence can be especially difficult for adolescents because they may
skip HIV medicine to hide their HIV-positive status from others.
D. “Many adolescents infected with HIV do not know that they are HIV positive.”

99. Nurse Isabel knows that Mark will not be subjected to any procedure without his
family’s written informed consent, EXCEPT, ______.

A. When disclosure of material information to patient will jeopardize the success of


treatment, in which case, third party disclosure and consent shall be in order
B. When Mark’s family does not waive its right in writing
C. When the patient is either a minor, or legally incompetent, in which case, a third
party consent is required
D. In emergency cases, when the patient is at imminent risk of physical injury

100. The MOST APPROPRIATE nursing diagnosis for Mark’s case who is an adolescent
is, which of the following nursing diagnoses?

A. Social Isolation
B. Compromised family coping
C. Anxiety related to the stigma of his admitting diagnosis.
D. Risk for compromised human dignity
NURSING PRACTICE III

Situation – You are employed as a staff-nurse in a tertiary government hospital in


Manila and had a training program with the Red Cross on Disaster Preparedness
Management before deciding to work back home in Cagayan province. One day, there
was a strong typhoon that hit your place and despite the typhoon, you decided to report
for duty.

1. You are assigned in the surgical ward with 10 patients and across your ward is the
Pediatric unit with 12 patients but their nurses did not report for duty. Your immediate
supervisor ordered you to cover both units temporarily. Which of the following
PRIORITY actions should you do in this situation?

A. Call the medical director to augment the staffing with more nurses.
B. Identify and cluster patients according to their needs
C. Request the watchers to assist you with patients on ventilators
D. Ask if you can go home early, so you can help your family too

2. Your supervisor is aware that you have a previous experience in the Pediatric unit.
She instructed you to take care of the children while she will take care of the surgical
patients. How should you respond to this?

A. Do not accept this responsibility as there is a possibility for you to commit a mistake
B. Accept this assignment as you have the competency to take care of the children
C. Do not accept this assignment as the Pediatric ward is in disarray and patients are
with varying illnesses
D. Argue with your supervisor that this is not correct as your nursing license will be at
stake
3. After a week the medical director has organized an assembly to recognize the health
care personnel who were on duty during the typhoon incident. You were given a
certificate of appreciation and a trip to Tagaytay for an advanced training on Disaster
and Preparedness program. This incentive is a form of

A. Personal and Research development


B. Quality improvement
C. Promotional activity
D. Personal and professional development

4. In line with your career pathway, you decided to enroll in the graduate program
leading to clinical specialization. You were offered to become a staff-development
coordinator by the chief nurse. What is your APPROPRIATE response to this offer?

A. Accept the position right away even without a written contract


B. Do not accept the offer as this will entail a lot of responsibilities and accountability
C. Talk to the chief nurse, and inform her you are still finishing your graduate studies
D. Accept the job as this is a promotion you have been waiting for

5. Management of resources is a core competency that must be developed by every


graduate nurse in the health care facility. Which of the following are the key elements
that will facilitate effective and efficient health care.

1. Workload of a staff
2. Financial resources
3. Positive practice environment
4. Proper functioning of equipment

A. 1, 2, 3 & 4
B. 1, 2 & 3
C. 1 & 3
D. 1 & 2

Situation – A patient with hyperthyroidism is admitted to your unit. She asks the nurse
about the endocrine functions.

6. To begin your discussion, you explain to her that the endocrine glands include; Which
of the following?

A. Pituitary, thyroid, parathyroids, adrenals, pancreatic islets, and hypothalamus


B. Pituitary, thyroid, parathyroids, pancreatic islets, ovaries, and testes
C. Pituitary, thyroid, parathyroids, adrenals, and pancreatic islets
D. Pituitary, thyroid, parathyroids, adrenals, pancreatic islets, ovaries and testes

7. The _____________is the link between the nervous system and the endocrine system.

A. Spinal cord
B. Hypothalamus
C. Pituitary gland
D. Thyroid gland

8. This endocrine disorder is a severe form of hypothyroidism characterized by an


accumulation of mucopolysaccharides in subcutaneous and other interstitial tissues.

A. Myxedema
B. Thyrotoxicosis
C. Cushing’s syndrome
D. Hashimoto’s disease

9. The patient asks you about goiter. You describe this disorder as

A. A condition produced by excessive endogenous or exogenous thyroid hormone


B. The enlargement of the thyroid gland and usually caused by an iodine-deficient diet
C. None of the choices
D. Inflammation of the thyroid gland that may lead to chronic hypothyroidism or resolve
spontaneously
10. The patient asks what is a normal thyroid state? The BEST response is

A. Euthyroid
B. Parathyroid
C. Hyperthyroid
D. Panthyroid

Situation – Ms. Simon is a nurse manager of a medical unit in a tertiary hospital. She is
responsible for the supervision of the staff nurses assigned in the unit and it her health
care personnel.

11. Ms. Simon reminds the nursing staff to safeguard and improve the quality of care
given to the patients. Which of the following activities is a PRIORITY to safeguard
nursing practice? Nurse should

A. Understand their professional, legal and ethical obligations and responsibilities


B. Know their strengths and weaknesses
C. Document care accurately
D. Practice nursing competently

12. Ms. Simon in collaboration with maintenance personnel prepares a safety program
which includes periodic inspections of electrical equipment, conduct of fire drills and
proper disposal of hazardous wastes. Which component of a quality program do these
activities illustrates?

A. Total quality program


B. Risk management program
C. Quality assurance program
D. Quality improvement program

13. Ms. Simon reviews a report on the number of patients who fell off from their beds
and other fall incidents. Which of the following is an APROPRIATE approach for Ms.
Simon to manage the situation?

A. Apply risk management principles


B. Implement a system approach
C. Implement quality assurance measure
D. Apply total quality management principles

14. A patient is admitted for episodes of seizures. Sometimes during the night, the
patient fell off from the bed. The night nurse found the patient on the floor. The nurse
manager instructs the night nurse to explain what happened. Which of the following
BEST describes an incident report? An incident report

1. Serves as a record of facts surrounding the event


2. Identifies the people involved, the date, time and location of the event
3. Is a part of the patient’s medical record
4. Can be used in a court of law

A. 1, 3, 4
B. 1, 2, 4
C. 2, 3, 4
D. 1, 2, 3, 4

15. Ms. Simon receives a report that a nurse administered a drug incorrectly to a
diabetic patient who need more than one type of insulin. The nurses action however did
not cause a negative effect on the patient. Which is a MOST appropriate action for the
nurse manager to take?

A. Report the nurse to the attending physician


B. Refer the nurse to the risk management committee of the hospital
C. Refer the nurse to the quality assurance committee of the hospital
D. Instruct the nurse to write an incident report
Situation – You are assigned to take care of patient Marko with brain tumor who has
been manifesting signs and symptoms of diabetes insipidus.

16. What should you watch for in this kind of disorder?

1. Excessive thirst
2. Excessive urination
3. Decreased weight
4. Craving for salt

A. 3 and 4
B. 2 and 3
C. 2, 3 and 4
D. 1, 2 and 3

17. In planning for care of a patient with Syndrome of Inappropriate Antidiuretic


Hormone (SIADH), you should keep in mind that Antidiuretic hormone (ADH) is a
deficiency of which of the following?

A. Thyroid stimulating hormone


B. Thyroxine
C. Vasopressin
D. Pituitary hormone

18. Which of the following is the PRIORITY goal of the health care team when a patient
is afflicted with this syndrome (SIADH)?

A. Documentation of urination pattern


B. Maintenance of fluid and electrolyte balance
C. Maintenance of glucose level
D. Adequate dietary supplement

19. When discharging a patient with diabetes insipidus, which of the following is the
BEST instruction to be given to the patient?

A. Carry the names of his attending physician


B. Written instruction on where to go for treatment
C. Carry his emergency meds
D. Wear an identification bracelet at all times

20. Diabetes insipidus is a possible complication of pituitary surgery. Which of the


following is an indication of this condition?

A. Urine output has glucose and ketone bodies


B. Urine specific gravity greater than 1.030
C. Urine output is 5 to 10 liters/day
D. Urine output exceed input

Situation – Mr. Rex, 36 years old, married and an architect has consulted the ER
because of on and off abdominal pain which bothers him a lot. Most of the time he has
irregular meals the past few months. After he was examined by the physician and
diagnostic tests were done on him, he was diagnosed to have duodenal ulcer

21. When Nurse Gladys is conducting an initial interview on Mr. Rex’s health
perception-health maintenance pattern related to gastro-intestinal function, which of
the following question is the MOST appropriate to ask?

A. What is your budget allowance for your daily meal?


B. What is your usual eating pattern?
C. Have you been to foreign countries lately?
D. Do you have diarrhea when you are under stress?

22. Nurse Gladys is aware that when a patient has Duodenal ulcer, he is likely to
present which ONE of these manifestations?

A. Presence of fullness of the abdomen even without food


B. Feeling of nausea and vomits whenever he has food intake
C. Pain is relieved immediately after he has eaten
D. Presence of pain even one hour after ingestion of food

23. The physician ordered bland diet to Mr. Rex. What kind of food do you expect to find
in his meal tray?

A. Pork steak, fried fish, white rice


B. Fried chicken, french fries, spaghetti
C. Pureed squash, creamed soup, white rice
D. Ground beef, mashed potato, white rice

24. As part of the medical treatment, to reduce the production of gastric secretion, an
anti-cholinergic drug was ordered. Which of the following enumerated drugs below
does NOT have this kind of action?

A. Riopan
B. Xanax
C. Maalox
D. Mylanta

25. Nurse Denky, is the night duty nurse of Mr. Rex who was given an endorsement by
Nurse Gladys to give the ordered Antacid every 2 hours. At 4’oclock in the morning, the
patient was fast asleep. What PRIORITY action should Nurse Denky undertake?

A. Ask your supervisor whether there is a need to give the drug or not
B. Skip the 4’oclock dose of antacid and give a double dose at 6 AM
C. Wake him up and administer the drug as ordered
D. Let him continue to sleep, just give the drug when he wakes up

Situation – Health care professionals have become increasingly aware that they need to
improve the way they have to communicate with patients and the public to avoid gaps
and misunderstanding.

26. A group of nursing students are attending a briefing from their faculty member on
the interviewing techniques when admitting a patient to the ward. Which of the
following techniques is used by the student nurse Astra if she uses EXACT words in
clarifying responses of the patient?

A. Reflecting
B. Paraphrasing
C. Exploring
D. Restating

27. Nurse Razl is taking care of an older person who is seated at the bedside waiting for
her food tray. How should the Nurse APPROPRIATELY communicate with the patient in
this situation?

A. Sit on the bed while conversing with her


B. Stand in front, with her hands on the pocket
C. Sit at the eye level with the patient
D. Touch her while waiting for the food tray

28. A nurse is caring for a cardiac patient whose younger son just passed away 3
months ago. She is crying when she was doing her morning rounds and said to her”
How could you possibly understand what I am going through”. Which of the following is
the BEST response of the nurse?

A. “Take time to relax with your friends and relatives by going with them to watch
movies.”
B. “You are right, I cannot really understand, but perhaps you can tell me more about
your feelings”.
C. “It takes time really to forget difficult events in our life but God will always be there
to guide us”.
D. “You don’t have to cry, there are other children in the family who will be with you”.
29. You are caring for Mr. RAD, a newly diagnosed diabetic patient who is preparing to
go home. His wife is not so familiar with his diagnosis so she asked you the primary
difference between type I and type II diabetes. Your APPROPRIATE response is

A. Type I diabetes has a strong disposition than type II


B. Type I diabetes is always insulin dependent while type 2 DM is not
C. Type II diabetes is more common in younger people than type I DM
D. Type II occurs frequently in men while type I occurs in women

30. Following a diagnostic work-up of a patient with mitral valve prolapse, the nurse
was asked by the mother what type of treatment is normally given to this type of
condition? The BEST response of the nurse is

A. “I will consult my supervisor and the resident they can explain better
B. “Usually, with my experience they are recommended for surgery”
C. “I don’t know, you can ask your physician when he visits
D. “Adequate rest, sleep and medication are sufficient if uncomplicated”

Situation – Ms. Lilia, 62 years old had undergone partial gastrectomy and is ready for
discharge. Nurse Chit provided the patient oral and written instructions with her
daughter around.

31. As post gastrectomized patient, which of the following is the APPROPRIATE


instruction to be provided to her in relation to wound and incision care?

A. Suturing of incision site if found open


B. Demonstration of proper wound management
C. Put anesthetic agent to incision site if with pain
D. Inspect for presence of purulent materials

32. After gastrectomy, dumping syndrome may occur, what warning sign has to be
emphasized to the patient and family members to be reported, EXCEPT

A. Dizziness and palpitation


B. Cramps and abdominal fullness
C. Nausea and vomiting
D. Vomiting of purulent and red blood

33. One of the symptomatic reliefs of Dumping syndrome is through diet management.
Which of the following are considered effective to be followed by the patient EXCEPT

A. Avoid taking liquids with meals


B. Assume recumbent position after meals
C. Eat a small but frequently feeding
D. Take liquids with meals at all times

34. Nurse Chit informed patient Lilia the possible risk of developing complications after
surgery. Which one of this can happen if she is not compliant with her health
instruction?

1. Aspiration pneumonia
2. Electrolyte imbalance
3. Wound infection
4. Atelectasis

A. 3 & 4
B. 1, 2, 3 & 4
C. 1, 2 & 3
D. 1 & 2

35. When a patient goes for Billroth II surgery of the stomach, it is very important that
patient and family members have to be oriented that this operation involves which of
the following?

A. Removal of the stomach anastomosed to the anal sphincter


B. Removal of the stomach anastomosed to the duodenum
C. Removal of the stomach anastomosed to the ileum
D. Removal of the stomach with the remaining segment anastomosed to the jejunum

Situation – The Quality Assurance team of a Tertiary hospital gave a report to the
management that the statistics for the month showed that medication error is still the
no 1 problem committed by the nurses. A competency enhancement program of the
nurses have been designed by the staff-development unit of the hospital on drug
administration.

36. Nurse Alyanah, is in charge of mentoring a graduate nurses, she started the session
on safety and quality in pharmacology and asked what errors in drug administration are
avoided in medication reconciliation is done in clinical setting? Which of the following
are the APPROPRIATE responses?

1. Omission
2. Duplications
3. Drug interaction
4. Dosing errors

A. 1, 2 & 4
B. 1 & 2
C. 1, 2, 3 & 4
D. 3 & 4

37. In drug administration, Nurse Alyanah emphasized with the group some of the
salient guidelines for the correct administration of drugs. Which of the following is NOT
advisable? The BEST response of Nurse Gigi, one of the attendees is

A. Check medication order with the doctor’s orders, kardex, medicine sheet against
medicine card
B. Administer only the drugs that you have prepared and not by another nurse in the
unit
C. Verify doses of drugs that are potentially toxic with another nurse or Pharmacist
D. Give first the drugs to a patient who needs extra assistance when 2 or 3 patients
have some schedule

38. Part of the mentoring group program is patient Ramon receiving ACE inhibitor
drugs to treat his hypertension. Nurse Helen was asked, “what should be included in
your assessment prior to the administration of this drug? Which of the following is her
BEST response EXCEPT

A. Baseline blood pressure


B. Persistent cough
C. Bowel elimination pattern
D. Sleeping resting pattern

39. Another question was asked to another attendee by Nurse Alyannah, “If patient
Ramon is given initial doses of ACE inhibitors, Captopril 25 mg 3x daily I hour before
meals together with Lasix. Which of the following adverse effects should be watched by
the nurse?

1. Hypotension
2. Dizziness
3. Tachycardia
4. Fainting

A. 1, 2, 3 & 4
B. 1 & 2
C. 1, 2 & 3
D. 3 & 4

40. The last item that was asked of the attendees is “When a physician ordered a drug
to a patient experiencing pain, which of the following list is NOT included as opiod
analgesic agent?

A. Methadone (dolphine)
B. Morphine sulfate
C. Fentanyl (sublimaze)
D. Lasix

Situation – Mr. Rod, 35 years old, an Engineer was rushed by his wife to the Emergency
room moaning because of severe pain of the left chest after carrying a heavy object. He
was examined by the physician and ordered to have a stat chest X-ray which revealed
that he has a spontaneous pneumothorax of the left lung. Patient was admitted for
further work-up and treatment.

41. In establishing a therapeutic nurse-patient relationship, which of the following is a


MOST appropriate nursing action when a patient is in pain?

A. Take the blood pressure, respiration and heart rate


B. Respect the verbalization of the patient about the pain
C. Request relatives for more information regarding the pain
D. Document at once the kind of pain the patient is presenting

42. As a nurse in charge of admitting the patient in the ward, which of the following is
your INITIAL step in pain assessment?

A. Conduct a focus assessment and later other parts of the body


B. Performs a comprehensive pain assessment on the patient
C. Observe patient’s responses while doing the assessment
D. Evaluate the intensity of pain and teach relaxation technique

43. In order to re expand the affected lung and eventually relieve the pain, Mr. Rod was
connected to a chest tube drainage. For a patient with Pneumothorax, the tube is
inserted at what part of the chest?

A. First intercostal space


B. Fourth or fifth intercostal space
C. Second or third intercostal space
D. Six intercostal space

44. After 6 hours of the insertion of the chest tube drainage, the patient could not sleep
because of severe pain. The patient was given Meperidine Hcl (demerol) 50 mg by IM
route. When a patient is receiving this drug, which of the following reactions should be
given PRIORITY attention by the nurse?

A. Nausea and vomiting


B. Respiratory depression
C. Clouded sensorium
D. Incontinence

45. Despite the pain medication, Mr. Rod still complains of severe pain. What should be
the nurse’s IMMEDIATE action of this concern?

A. Reassure the patient that an alternative drug will be given


B. Check vital signs and call the physician at once
C. Reassess the tolerance of the patient to the drug
D. Hold the next medication until the physician’s evaluation

Situation – Mrs. Abby, 69 year old was admitted in the medical unit because of
malnutrition. She had been anorexic for the last 6 months due to family problems. She
always feel fatigue with body weakness and have lost weight (12 lbs in 2 months) due to
poor appetite.

46. As the nurse-in-charge of this patient, if an individual has a caloric deficiency in the
diet, what specific manifestation do you expect to be present? EXCEPT

A. Loss of subcutaneous fat


B. Sparse hair
C. Muscle wasting
D. Listlessness
47. In order to improve the nutritional needs of patient Abby, the physician ordered
insertion of nasogastric tube (NGT). What is the APPROPRIATE size of the feeding tube
you should prepare for this purpose?

A. Fr. 13 to 14
B. Fr. 15 to 16
C. Fr. 16 to 18
D. Fr. 8 to 12

48. When inserting NGT to patient Abby, the SAFETY ALERT and decision points you
should implement are the following

1. Identify the patient using name and birthday


2. Determine length of the tube to be inserted & mark location
3. Position her at 45 degrees head of bed elevation
4. Let her mouth breathe and swallow during the procedure
5. Measure gastric residual volume during tube feeding

A. 1, 2, 3, 4
B. 1, 2, 3, 4 & 5
C. 2 & 3
D. 1 & 2

49. After inserting the NGT to patient Abby, you would like to be sure the tube is in the
stomach. Which of the following PH VALUE of the gastric aspirate indicates it is in
place?

A. 6
B. 7
C. 8
D. 5

50. While you were feeding patient Abby, you observed that the tube is clogged. Which
of the following solutions is APPROPRIATE to use in flushing the NGT?

A. 22 to 25 ml ice H20
B. 20 to 30 ml of NSS
C. 18 to 25 ml lukewarm H20
D. 15 to 20 ml of distilled H20

Situation – To ensure continuity of care, the nurse should be able to document relevant
baseline information about patient condition.

51. Which assessment findings will the nurse record to be consistent in a client with
respiratory failure?

A. Hypoxemia, hypercapnia
B. Hyperventilation
C. Respiratory alkalosis
D. Hypoxia, hypocapnia

52. Oxygen therapy is the administration of oxygen at a concentration greater than that
found in the environmental atmosphere. The goal of oxygen therapy is to

A. Provide adequate transport of oxygen in the blood while decreasing the work of
breathing and reducing stress on the myocardium
B. Reverse the effects of hypoxemia in a patient
C. None of these
D. Deliver high concentration of oxygen in the blood while decreasing the work of
breathing and reducing hypoxemia on the myocardium

53. The nurse is aware of the differences between hypoxia and hypoxemia. Hypoxemia
is characterized by

A. A deficiency of oxygen and the biotic environment


B. A decrease in oxygen supply to the tissues
C. An insufficient amount of oxygen in the body and, if severe enough, can be life-
threatening
D. A decrease in the arterial oxygen tension in the blood and is manifested by changes
in mental status (progressing through impaired judgement, agitation, disorientation,
confusion, lethargy, and coma)

54. A nurse suspects a diabetic patient to have hypokalemia when she observed which
of the following symptoms to be documented?

A. Sunken eyeballs, Kausmaul breathing, hunger


B. Apathy, weakness, abdominal distention
C. Edema, bounding pulse, confusion
D. Spams, hypotension, convulsion

55. In the initial presentation of type 1 diabetes mellitus in a patient. Which symptoms
would the nurse expect to assess and document?

A. Polydipsia and polyphagia


B. Short attention span and hyperactivity
C. Blurred vision and weight gain
D. Weight loss and hypotonic reflexes

Situation – You have been assigned as the new pain management nurse in your hospital.
You receive various types of patients with different types of pain during your shift today.

56. You assessed a 67 year-old patient for reports of episodic, sudden onset, right-sided
facial pain. The patient describes the pain as fleeting, electric-like and triggered by
light touch and brushing of the teeth. You suspect

A. Temporomandibular disorder
B. Trigeminal Neuralgia
C. Myofascial pain syndrome
D. Facet syndrome

57. Which of the following is the recommended protocol for preventing constipation
when starting a patient on opiods?

A. Increasing fluids and exercise


B. Using a bowel stimulant and stool softener
C. Adding bulk fiber to the diet
D. Giving the patient enemas as needed

58. A 12-year old patient who is receiving in-home care without IV access needs
medication for breakthrough pain. Which of the following is the most effective route of
administration?

A. Nebulized
B. Oral-transmucosal
C. Intranasal
D. Transdermal

59. You have some older patients in your department. Which non pharmacologic
intervention is difficult to use with older adults who are cognitively impaired?

A. Distraction
B. Heat application
C. Guided imagery
D. Aromatherpy

60. You assess a patient with complex regional pain syndrome. The nurse is concerned
about the patient’s depressed mood, because she or he has said: “I can’t live with this
pain”. You further assess for suicide risk, because

A. Suicidal thoughts are common in patients with chronic pain


B. Decreased pain threshold lead to suicidal thoughts
C. Verbalization of suicidal thoughts is a way for patients to get attention
D. Suicidal thoughts are often expressed by patients with acute pain

Situation – Nurse Belen is assigned in the medical unit and caring for patients mostly
affected by Endocrine disorders. As a professional nurse, she is expected to be
competent in taking care of them.

61. The endocrine system is a glandular tissue that produce, store and secretes
hormone. Which of the following is UNIQUE characteristics to all hormones?

A. Circulate into the blood for metabolic processes


B. Enter to cell to alter genetic development
C. Accelerate the anabolism and catabolism regulation
D. Influence cellular activity of specific target tissues

62. When conducting a physical assessment of patients with endocrine disorders, the
nurse is guided that the ONLY endocrine organ that can done by palpation is the

A. Parathyroid gland
B. Adrenal Gland
C. Pituitary gland
D. Thyroid Gland

63. Nurse Belen is taking care of Ms. Flora, 32-year-old, who has a tentative diagnosis
of thyrotoxicosis. With the increase thyroid hormone that stimulates metabolic rate in
this condition, which of the following is NOT expected by the patient to manifest?

A. Increase cardiac output


B. Hypermotile bowels and diarrhea
C. Palpitation
D. Cold Intolerance

64. The attending physician of Ms. Flora ordered several laboratory and diagnostic test.
She was ordered to undergo thyroid scanning. Which of the following data should be
obtained by nurse Belen PRIOR to the procedure? Check if patient is _______________.

1. allergic to iodine
2. allergic to shellfish
3. pregnancy
4. taking anti-thyroid drug

A. 1,2,3 & 4
B. 1 & 3
C. 1,2 & 4
D. 1 & 2

65. Patient Flora is for discharge in 3 days time. In the discharge plan of Nurse Belen,
she has identified a priority nursing diagnosis which is, "Risk to Impaired vision related
to inability to close the eyelids completely". Which of the following interventions is NOT
APPROPRIATE to protect the eye of the patient from injury and maintain visual acuity?
Use _______________.

A. Tinted eyeglasses as protective mechanism to bright light.


B. Artificial tears as need to moisten the dry eyes.
C. Clear eyeglasses when doing outdoor activities
D. Report to the health care professionals any pain or visual changes.

Situation - Nurse Gina is on 7-3 shift in the ER. The unit is crowded with patients and
relative. She is attending to two patients, Roy having chest pains and Rey with severe
abdominal pains. The doctor ordered Nitroglycerine 0.5mg sublingually for Rex but the
nurse mistakenly administered the drug to Rey.

66. The Nurse is liable to be charged with _________.

A. Tort
B. Negligence
C. maleficence
D. Non-maleficence

67. Helen, a newly hired nurse was pulled out from the Pediatric Unit by the Supervisor
and assigned to surgical ward temporarily for the shift at four o'clock in the afternoon.
She has to feed a post-operative patient who had a tracheostomy and nasogastric tube
(NGT) as well. The feeding was mistakenly infused to the tracheostomy instead of
infusing it to the NGT. Helen can be liable for _________________.

A. Assault
B. Malpractice
C. Tort
D. Negligence

68. Rene is a staff Nurse who was assigned in the medical ward. The physician ordered
20 units of insulin injection to a client with diabetes. Nurse Rena administered 20cc of
insulin instead. After 30 minutes the client developed hypoglycemia and died. Nurse is
liable for _______________.

A. Negligence
B. Tort
C. Malpractice
D. Assault

69. A staff nurse was caught by her supervisor administering high doses of morphine to
a patient with leukemia for fee of 25,000 thousand pesos given by the mother. The
nurse liable for ______________.

A. Assault
B. Tort
C. Bribery
D. Malpractice

70. Nurse Cleo is assigned to a cranky and demanding patient suffering from a
peripheral neuropathy. One day the patient was chilling and needed blankets and a hot
water bag (HWB). Nurse Cleo places the HWB without cover and placed it directly to
the patient's foot. The patient suffered 2nd degree burns. Nurse Cleo may be sued by
relatives for __________.

A. Breach of duty
B. Negligence
C. Justice
D. Malpractice

Situation - Ms. CD, 15 year old, 100lbs, 5ft tall, admitted to your unit for diagnostic
evaluation and nutritional support. She is moderately dehydrated with potassium level
of 2.3mEq/L. She has experienced weight loss of more than 15 percent within the past 3
months.

71. What is the PRIMARY Collaborative goal of treatment for Ms. CD?

A. Resolve possible dysfunctional family roles as an adolescent.


B. Increase strong desire to eat.
C. Restore normal nutrition and weight.
D. Assist her to increase feelings of control over eating.

72. For Ms. CD which route for delivery of nutrition and fluids will the health care team
try FIRST?

A. Gastrostomy tube.
B. Oral route
C. Nasogastric route
D. Intravenous route

73. In caring for this patient suffering from anorexia nervosa, which task can be
delegated to the nursing assistant?
A. Obtaining special food for the patient when she request it.
B. Sitting with the patient during meals and for about an hour after meal.
C. Weighing the patient daily and reinforcing that she is underweight.
D. Observing for the reporting ritualistic behaviors related to food.

74. You find Ms. CD in her room running and trotting around for about the last 30
minutes. What is the BEST response to give her at this time?

A. “Tell me why you are running and trotting around the room?”
B. “Stop running right now, otherwise, we will change your exercise program.”
C. “We have talked about exercise and agreed to reach your weight goal first.”
D. “If you continue to exercise like this, you are going to eat more.”

75. Ms. CD is at risk for refeeding syndrome that is caused by rapid feeding. What
should be the PRIORITY action of the health care team to prevent complications
associated with this syndrome?

A. Assess for signs of pallor of the extremities and sluggish capillary refill.
B. Monitor for decreased bowel sounds, nausea, bloating, and abdominal distention.
C. Observe for signs of secret purging and ingestion of water to increase weight.
D. Monitor for peripheral edema, crackles in the lungs, and jugular vein distention.

Situation - Mina, a cashier in a bookstore consulted the ER because of abdominal


cramps, nausea and vomiting and slight fever. She has been experiencing these for
almost 3 days now. After a complete physical examination, laboratory and diagnostic
tests, the physician ordered her to be admitted for Exploratory laparotomy, you are the
Nurse-in-charge of this patient.

76. As a nurse, the responsibility for your pre-operative care for patient.
Nina includes which of the following EXCEPT:

A. Providing adequate nutrition and elimination


B. Preparing the operative site of the abdominal region.
C. Explaining the surgical procedure while admitting the patient.
D. Ensuring the patient is psychologically ready for the surgery.

77. Which of the following pre-operative medications do you expect to be ordered by the
surgeon the night before surgery?

A. Morphine Sulfate
B. Phenergan
C. Valium
D. Demerol

78. The patient is scheduled for operation at 8 'o'clock. In the morning, the pre-
operative medications were administered an hour before the surgery. What PRIORITY
nursing measures should you perform before transferring the patient to the operating
room?

A. Ask her to sign the consent.


B. Check if she is using her identification bracelet.
C. Assist the patient to urinate.
D. Determine what type of anesthesia will be done.

79. What medicines is ordered by the surgeon to reduce salivation and bronchial
secretions before the operations?

A. Benadryl
B. Atropine Sulfate
C. Magnesium SO4
D. Codeine

80. After 36 hours post exploratory laparotomy, which of the following PRIORITY
physical findings should be reported immediately to the attending physician?

A. Presence of flatus
B. No bowel sounds
C. Rigid Abdomen
D. Nausea and Vomiting

Situation - A nasogastric tube (NGT) was ordered by the Emergency physician to a male
patient suffering from head trauma after undergoing thorough physical examinations
and diagnostic tests. The physician diagnosed him to have a basilar skull fracture.

81. What PRIORITY nursing action should be done to the patient considering his
present condition?

A. Test the gastric content for the blood.


B. Attempt to place the tube into the duodenum.
C. Check patency of airway before nasogastric tube insertion.
D. Use extra lubrication when inserting the nasogastric tube.

82. Another patient is in need of nasogastric tube. The nurse, while inserting the tube,
observes that the patient started to cough and shows difficulty of breathing. Which of
the following is the BEST nursing action to be done in this situation?

A. Quickly remove the tube and let the patient rest.


B. Remove the tube and reinsert when difficulty of breathing subsides.
C. Notify the doctor at once and inform him of your observation.
D. Pull back the tube and wait for further order.

83. Nurse Ria is assessing the correct placement of the tube. Aspiration of the stomach
contents as well as checking for PH. Which PH value result confirms the placement of
the tube is in place?

A. 6.0
B. 7.35
C. 6.50
D. 5.2

84. What is the recommended position during NGT insertion?

A. Supine Position
B. High Fowler’s
C. Semi-Fowler position
D. Low Fowler’s

85. Nurse Ria is preparing to remove the NGT from the patient. Which of the following
PRIORITY instruction should be given to the patient before she removes the tube?

A. Exhales and rest for a while.


B. Inhale and exhale quickly.
C. Take and hold a deep breath.
D. Take and hold a deep breath.

Situation - You are assigned in the PACU and is currently caring for patients undergoing
different types of surgical operations.

86. Nursing interventions indicated during the patient’s recovery from the general
anesthesia in the PACU includes which of the following?

A. Encouraging deep breathing and coughing.


B. Withholding analgesics until the patient is discharged from the PACU.
C. Placing the patients in supine position.
D. Restraining patients during episodes of emergence delirium

87. Post-operative nausea and vomiting present the greatest risk for?

A. An 81-year-old, 55 kg woman following cystoscopy under local anesthesia.


B. A 45-year-old 70kg. Man following a diagnostic arthroscopy under epidural
anesthesia.
C. A 23-year-old, 125kg woman following a diagnostic laparoscopy under general
anesthesia.
D. A 14-year-old, 40kg boy following an orchiopexy under general anesthesia.

88. In preparation for discharge after surgery, which of the following should the nurse
advise the patient?

A. The rationale for abstinence from sexual intercourse for 4 to 6 weeks.


B. A time frame for when various physical activities can be required.
C. The necessity of a referral to a nutritional center for management of dietary
restrictions.
D. The need to call the hospital clinical unit to report any abnormal signs and
symptoms.

89. Which of the following nursing actions during the immediate postoperative period
has the HIGHEST priority?

A. Observing for hemorrhage.


B. Maintaining a patent airway.
C. Recording intake and output.
D. Checking vital signs every 15 minutes

90. An informed consent is required for EXCEPT?

A. Irrigation of the external ear canal.


B. Closed reduction of a fracture.
C. Urethral Catheterization.
D. Insertion of an intravenous catheter.

Situation - Mr. Andy, 76 years old, a former chairman of the Board of trustees of an
academic institution had a severe chest pain and was rushed to the medical center by
the member of the family. He was seen at the ER and was advised by the physician to be
admitted.

91. After a comprehensive assessment and diagnostic tests, he was found out to have
blockage of 2 coronary arteries. The wife and his 2 children, a nurse and a doctor was
trying to convince their father to undergo open heart surgery to relieve his discomforts.
In this ethical decision who should be followed?

A. wife
B. children
C. Patient
D. Doctor

92. When Mr. Andy decides not to go for surgery and respected by the physician, which
ethical principle is being observed?

A. Justice
B. Beneficence
C. Autonomy
D. Non-Maleficence

93. While in the hospital, Mr. Andy keeps on asking for pain medications. When the
Nurse of Mr. Andy offers his pain medication when needed, she is applying the ethical
principle of?

A. Fidelity
B. Beneficence
C. Advocacy
D. Justice

94. The Nurse observed that the dose of the PRN pain medication ordered by the
physician to Mr. Andy is quite high for his age, so she suggested for lower dose. What
ethical behavior is being manifested by the nurse with this action?

A. Fidelity
B. Nonmalifecence
C. Advocacy
D. Justice

95. Mr. Andy is on bed rest. When he uses the call bell for the nurse to assist him to be
transferred to the commode and answered him to wait for 5 minutes and came on time
is an example of what ethical principle?

A. Maleficence
B. Non-maleficence
C. Fidelity
D. Advocacy

Situation - Nurse Rodilyn, conducts a research study on all male stroke patients
confined in the hospital for a week. Her participants must be able to relate their life
experiences as stroke patients.

96. Nurse Rodilyn selects only patients who have suffered stroke admitted and
discharged from the hospital as participants of the study. Which of the following
describes the selection of participants of Nurse Rodilyn in her study?

A. Random Sampling
B. Experimental Sampling
C. Purposive Sampling
D. Stratified Sampling

97. Nurse Rodilyn respondents were asked to sign consent before the conduct of the
study. Three (3) participants were forced to join the study. Which right is violated when
they are forced to become research respondent?

A. Right to full disclosure


B. Right to self determination
C. Right of privacy and confidentiality
D. Right not to be harmed.

98. One respondents wants to withdraw from the study, which ethical principle in
research should Nurse Rodilyn observe?

A. Refused request to withdraw as participant.


B. Allow participant to withdraw anytime.
C. Provide incentive not to withdraw.
D. Remind him of his duty as participant.

99. Nurse Rodilyn conducted a series of interview with respondents on different times
and occasion. This is to determine their experience and differentiate responses to
questions. This method is done in which of the following?

A. Experimental study
B. Qualitative study
C. Historical study
D. Quantitative study

100. Before Nurse Rodilyn analyzed the time, budget, and materials needed to complete
the research project. This undertaking in research is a component
of____.

A. Researchability
B. Reliability
C. Feasibility
D. Validity

NURSING PRATICE IV

Situation - Nurse Stephanie just finished emergency and disaster training for one month
in the Disaster and Risk Reduction Management conducted by the Department of
Health.
1. To Nurse Stephanie’s knowledge, the key difference between emergencies and
disaster is that:

A. Emergencies can typically be handled by avoidable emergency services.


B. Disaster results from man-made error.
C. Disasters typically involve local emergency services and no other agencies.
D. Emergencies are controlled.

2. Stephanie understands that emergency is any actual threat to public safety or public
health. Which of the following is NOT applicable?

A. “Hostage crisis. In Jolo Sulu last December 2018


B. “I cannot find my mobile phone”.
C. “My son has not come home from school”.
D. “A plane is about to crash”.

3. During an earthquake, the impact is greatest at areas close to the epicenter. Injuries
arise primarily from falling objects and collapsing buildings. Direct impact on health
includes high mortality from severe crash injuries. Stephanie arrives at the scene as the
first responder. Which action is MOST appropriate to assure safety?

A. Clear the walking wounded using verbal instructions. “If you can walk, move over
there”
B. Give priority to dying victims.
C. Tag patients
D. Stop at each victim and quickly assess their respiration.

4. Cardiopulmonary resuscitation maybe given to the victims. How many Seconds will
Stephanie consider for perfusion to be present?

A. Ten seconds
B. One second
C. Five seconds
D. Two seconds

5. A group of passengers enters the emergency room with complaints of cough,


tightness in the throat, and extreme periorbital swelling. There is a strong odor that
exudes from their clothes. They report exposure to “bomb that was placed in the bus
terminal. What is the PRIORITY action of the ER nurse?

A. Direct clients to the clean zone for immediate treatment.


B. Assist clients in the decontamination area.
C. Readily transfer patients and visitors from the area.
D. Check vital signs and auscultate lung sounds.

Situation - Nurse Anne is part of the emergency team of hospital x. She collaborates
with other members of the health to Provide quality care to patients

6. Nurse Anne understands that collaborative interventions are therapies that require
the participation of health professionals. Which of the following personnel participates
in collaborative efforts? The______

A. Physician and the nurses intervention


B. patient and the nurse's intervention
C. patient and the physicians intervention
D. multiple health care professionals

7. Nurse Anne understands that health care professionals have a role in collaboration.
Which of the following statements BEST Explains the role of the nurse in the plan of
care for patients? The nurse_____

A. collaborates with colleagues and the patient’s Family to provide combined expertise
in planning care
B. works independently to plan and deliver care and should not depend on other staff
members for assistance
C. depends on the latest literature to complete and excellent plan of care for patients
D. consults the physician for direction in establishing goals for the patients

8. In one of her shifts, Nurse Anne admits a 35 year old female for head injury sustained
in a car accident. Nurse Anne notifies the neurosurgeon. The later assesses the
condition of the patient using the objective measurement of three essential components
of a neurological examination. Which of the following responses is Not an essential
component?

A. Best non-verbal response


B. Best motor response
C. Spontaneity of eye opening
D. Best verbal response

9. The neurosurgeon determines, that the patient has increasing intracranial pressure.
Which of the following interventions would be MOST appropriate for nurse Anne to
perform? She should

A. provide a quiet and brightly lit environment


B. elevate the head 15 to 30 degrees
C. encourage the victim to drink clear fluids
D. teach controlled coughing and deep breathing

10. The patient was assessed to have a Glascow Coma Scale score of 3. In order to
protect the airway, nurse Anne collaborates with the health care team to assist with
which of the following?

A. Endotracheal intubation.
B. Assessment of bowel sounds.
C. Questioning the patient regarding difficulty of breathing.
D. Monitoring oxygen saturation.

Situation – Nurse Ditas cares for a male patient with HIV. She conducts a study on HIV.

11. Nurse Ditas wants to find out the lived experiences of a patient with HIV. This is an
example of phenomenology as a:

A. phenomenon theory
B. concept
C. Research design
D. philosophy

12. A characteristic or phenomenological study in for the researcher to understand the


lived experience from the point of view of the

A. Subject
B. close family member
C. nurse
D. researcher

13. A process in phenomenology where the researcher takes into account his/her own
beliefs and feelings Identity which he/she expect to discover and deliberately puts these
ideas aside is called:

A. recursive abstraction
B. Saturation
C. bracketing
D. coding

14. Nurse Ditas interviews her patient. She observes that he is hostile and does not
respond openly towards her questions. This is known as ____effect.

A. Hawthorne
B. Rosenthal
C. phenomenological
D. experimenter
15. Nurse Ditas collects data for the research. Which is the MOST common technique of
analyzing qualitative data?

A. statistical analysis
B. content analysis
C. participant impression
D. observer Impression

Situation – Mark a 63-year old had been suffering from heart failure for 3years He had
been advised by his cardiologist for a pacemaker to help his heartbeat which is – slow.

16. Mark asked Nurse Julia what it is that he will expect before, during and after the
procedure. Which among the health teachings is NOT an appropriate response?

A. Your doctor will recommend that you avoid vigorous exercise and heavy lifting for
about a month.
B. You will likely be awake during the surgery to implant the pacemaker and you will
have IV line for your medication.
C. Insulated wires will be inserted into the minor veins, and will be guide using X-ray
images.
D. You’ll stay a day in the hospital after having a pacemaker implanted.

17. Before the cardiologist decided Patient Mark to have his pacemaker implanted,
which one of the following tests is NOT done to find the cause of his irregular heart
beat?

A. Holter Monitoring and Smartwatch


B. electrocardiogram and echocardiogram
C. Stress Tests
D. SGOT and SGPT

18. Patient Mark asked if he can travel by plane with his implantable cardioverter
defibrillator (ICD). Which of the following is an INAPPROPRIATE response of Nurse
Julia?

A. “Be sure you” have ID card with you”


B. “Yes you can travel by airplane if cleared by your Cardiologist”
C. “Let the airport security staff know you have A pacemaker.”
D. “You can travel immediately after the pacemaker is implanted.”

19. Patient Mark asked further “Do cellphones interfere with pacemaker? Nurse Julia’s
response are the following EXCEPT

A. “Use the phone on the opposite ear “


B. “Its best to avoid keeping cellphones in your breast pockets on the side of the
device”
C. “cellphone less than watts do not appear to interfere with pacemaker”
D. “cellphone more than 3 watts do appear to interfere with pacemaker

20. Nurse Julia health instructions to patient Mark is to contact the doctor immediately
if the following sign occur. Which one is not priority?

A. Weight gain and swelling of the ankles


B. Fainting and dizzy spells
C. Frequent urination
D. Difficulty in breathing

Situation - Nurse Ceny assists in the care of several male patients with fluid volume
deficit.

21. Nurse Ceny reads the medical diagnoses of the patients assigned to her which
patient with the following medical conditions is at risk for fluid volume deficit? A patient
with

A. Congestive Heart Failure


B. Chronic Obstructive Pulmonary Disease
C. Cirrhosis
D. Colostomy

22. Nurse Ceny assesses the skin turgor on a 35 year old male patient. Which of the
following is an indicator of fluid volume deficit?

A. There is skin tenting.


B. The skin immediately flattens after release.
C. The skin is dry and pale.
D. The skin returns to a normal position within 1 or 2 seconds after pinching and lifting
the skin.

23. The physician prescribes an intravenous infusion of 750 ml Normal Saline to a 78-
year old male patient with fluid volume deficit and urinary tract infection. After a rapid
IV infusion, the patient begins to cough. The nurse raises the head part of the bed the
patient’s breathing. Nurse Ceny observes that the patient’s jugular veins distended and
respiratory rate is increased. Which of the following conditions is the patient
experiencing?

A. Beginning hypotonic water intoxication


B. Worsening of fluid volume deficit
C. Developing hypervolemia
D. Respiratory compromise due to ascites

24. Nurse Ceny inspects the tongue and oral mucosa of a 40-year old patient and notes
that they are dry. The patient had cough, fever. Nausea and vomiting which started
three days before he was admitted to the medical unit. The patient further complains of
feeling weak and dizzy. Which vital sign measurement would provide the BEST indicator
of the patient’s current fluid status?

A. Rate and depth of respiration


B. Body temperature
C. Pulse oximetry reading at rest
D. Blood pressure and pulse taken in lying and standing positions

25. Nurse Ceny assesses a 50-year old made with live failure due to cirrhosis. The
patient complains of a distended abdomen and dizziness, upon standing. The patient
looks pale, has weak radial pulse, and with delayed hand vein filling. Based on the
assessment data, Nurse Ceny writes a nursing diagnosis. Which of the following is the
MOST appropriate nursing diagnosis? Fluid volume

A. Deficit related to hormonal disturbances


B. deficit related to third space tied shifts
C. excess related to hormonal disturbances
D. excess related to third space fluid shifts

Situation - The nurse cares for teenage male with allergic rhinitis

26. Which of the following clinical manifestations would the nurse expect to find in the
client with rhinitis?

A. Nasal congestion, rhinorrhea and sneezing


B. Nasal congestion, pyrexic and rhinorrhea
C. Headache sore throat and sneezing
D. Nasal congestion, headache and sore throat

27. The physician prescribes decongestant intranasal spray. They instructs the client on
the proper use of the pay. Which of the following procedures is the CORRECT method

A. Finish installation of spray into one nostril before


B. Inhale quickly to prevent irritation of the mucous membranes
C. Blow the nose after spraying to prevent medications from entering the throat
D. Tilt the head slightly forward and angle the bottle toward the side of the nostril
28. The client asks the nurse about using decongestant nasal spray which of the
following statements by the nurse is CORRECT regarding its use?

A. “nasal sprays should be used sparingly because they can exacerbate a cough.”
B. “It would take several days of using nasal sprays before any effect on congestion is
noted”
C. “Nasal sprays should not be used for more than three days because it can worsen
congestion”
D. “They should be used along with corticosteroid nasal sprays for maximum benefit.”

29. The nurse would determine that her teaching goal on the use of a decongestant
nasal spray has been met when the client says

A. The spray should be used round-the-clock at equally spaced intervals.


B. “Nasal sprays must be combined with an oral antihistamine to achieve relief.”
C. overuse can result in nosebleeds and mucosal ulceration
D “Rebound rhinitis (rhinitis medicamentosa) is common with continued use”

30. The physician orders cromolyn sodium (nasal crom) for the client. The nurse
instructs the client that the most effective administration schedule is

A. when manifestations peak only, with two or three doses per day
B. at the start of the allergy season with a dose of once-a-day
C. one week before the allergy season begins with four to six doses per day
D. just after manifestations begin with a dose of twice-a-day

Situation – Ms. Simon newly registered nurse. She upgrades her nursing competencies
by attending seminars and workshops on advanced Nursing procedures. A workshop
she recently participated is on Physical as assessment. She applies her skills in the
female medical wait where she is assigned

31. Ms. Sinon inspects the abdomen of patient which of the following sequences
represent the order assessing: patient’s abdomen?

A. Palpate, percuss, auscultate, observe


B. Observe, auscultate, percuss, Palpate
C. Percuss, palpate, auscultate, observe
D. auscultate, observe, percuss, palpate

32. Ms. Simon auscultate for breath sounds. What type of data should auscultation
produce?

A Secondary
B. Subjective
C. Primary
D. Objective

33. Ms. Simon proceeds to palpate a patient’s body to detect warmth. What part of her
hand should she use?

A Finger tips
B. Back or dorsal surface
C. Ulnar surface
D. Finger pads

34. Ms. Simon assesses a patient for gag reflex. Which part of the same should she
place the tongue blade?

A. On the middle of the tongue and ask the patient to cough


B. On the uvula
C. Lightly on the posterior aspect of the tongue
D. On the front of the tongue and ask the patient to say “ahh”

35. Ms. Simon takes and records the body temperature of a patients the temperature
registers a reading of 38°C. Which of the following conditions will the patient MOST
likely demonstrate?
A. Increased pulse rate
B. Precordial pain
C. Dyspnea
D. Elevated blood pressure

Situation – You are a nurse manager of a female and male medical unit. You realize that
being a manager entails a great responsibility. One of your major responsibilities is
implement quality improvement initiatives in our unit.

36. You collaborate with the maintenance personnel to prepare a safety program. The
program includes periodic inspection of electrical equipment, conduct of tire drills and
proper disposal of hazardous waste materials. These activities illustrate a component of
which quality program?

A. Total quality management


B. Quality assurance
C. Risk management
D. Quality improvement

37. You call for a staff meeting. You remind the nurses to improve the quality of care to
patients. Which of the following activities is MOST important to safeguard nursing
practice?

A. Know own strengths and limitations.


B. Provide nursing care competently and efficiently.
C. Document care accurately.
D. Understand professional, legal, ethical obligations and responsibilities.

38. You take time to review reports for your staff. One report records the Incidence of
patients falling off from their beds and other falls, which of the following approaches is
MOST appropriate to manage the situation

A. Implement quality assurance measures.


B. Apply risk management principles.
C. Apply total quality management principles in the unit.
D. Implement a systems approach to the situation.

39. A staff nurse in your unit administers a drug incorrectly to a diabetic patient who
needs more than one type of insulin. The staff nurse’s action causes serious negative
effects on the patient. What would be the MOST appropriate action that you as the
nurse manager should take?

A. Refer the state nurse to the quality assurance committee


B. Refer the staff nurse to the risk management committee
C. Refer the patient to the attending physician immediately.
D. Ask the staff nurse to write a detailed incident report.

40. A patient in the unit had episodes of seizures. Sometime during the Patient on the
floor. As a nurse manager: you ask the nurse involved in finds the Incident to write an
incident report. Which of the following statements is NOT true about incident reports?
The report

A. is a part of the patient’s medical record


B. can be used in a court of law
C. identifies the people involved, the date, time and location of the incident
D. serves as a record of facts surrounding the incident

Situation – Nurse Gloria is assigned in the emergency department and takes care of
patients with different cases.

41. A client with multiple injuries is rushed after head on car collision. Which
assessment Nurse Gloria takes priority?

A. Unequal pupils
B. Irregular apical pulse
C. A deviated trachea
D. Ecchymosis in the flank area

42. The ambulance has transported a male client with severe chest pain. As the client is
being transferred to the emergency stretcher Gloria noted that the client is
unresponsive not breathing and no palpable pulse. Which of the following emergency
measures will Nurse Gloria anticipates to do?

A. Starting bag valve mask ventilation.


B. Performing chest compressions
C. Placing the defibrillator pads
D. Aiding with oral intubation

43. A child with fever has been admitted to the emergency department for several
hours. Cooling measures are ordered by the physician in order for the temperature to
come down. Which task would be appropriate for Nurse Gloria to do?

A. Educate the need for giving cool fluids


B. Prepare and administer a tepid bath
c. Tell the parent to use acetaminophen instead of aspirin.
D. Assist the child in removing out garments

44. When Gloria conducts primary assessment on a trauma patient. Which of the
following is considered one of the priority elements?

A. Complete vital signs


B. Brief neurologic assessment
C. Initiation of pulse oximetry
D. client's allergy history

45. Triage is the process of determining the priority of patients treatments based on the
severity of their condition. The purpose of reverse triage is to

A. Save those persons who are in the most critical condition.


B. Save scarce resources for future use
C. Do the greatest good for the greatest number with limited resources
D. The first responders on the tiring classification category

Situation - The nurse manager of the Oncology Unit Invites an oncology nurse specialist
to conduct an educational session to the nursing staff on the topic Hodgkin’s disease.

46. The nurse specialist explains the disease, which of the following is not a
characteristic of Hodgkin’s Characteristics of Hodgkin’s disease?

A. There is presence of Reed-Sternberg cells.


B. The disease occurs most often in the older adult.
C. The lymph nodes, spleen, and lives are involved.
D. The prognosis depends on the stage of the disease.

47. The nurse specialist describes the stages of Hodgkin’s disease. Which of the
following symptoms is MOST commonly an EARLY Indicator of Stage I?

A. Chest and back pains


B. subnormal body temperature
C. Unexplained fever, night sweats
D. swelling of extremities

48. The nurse specialist emphasizes the importance of preventing complications. Which
of the following conditions are complications of Hodgkin’s disease?

1. Nausea
2. Myocardial infarction
3. Anemia
4. Infection
5. Hypotension
A. 1, 2, 3, 5
B. 1, 2, 3, 4, 5
C. 1, 3, 4
D. 2, 3, 4, 5

49. The nurse specialist cites à situation. It a patient experiences episodes of severe
nausea and vomiting with more than 1000 ml. Of vomitus within a period of four hours,
which of the following is the nurses’ MOST appropriate action?

A. Withhold fluids for four hours.


B. Notify the physician
C. Observe the patient for another four hours
D. Place the patient on a liquid diet

50. The nurse specialist explain that chemotherapy is extremely toxic to the bone
marrow and the patient may develop thrombocytopenia. What is the PRIORITY goal of
the nurse? To take precautions to control

A. Bleeding
B. Infection
C. Hypotension
D. Diarrhea

Situation – Nurse Clara assists in the care of a male patient who has developed acute
respiratory acidosis.

51. Nurse Clara recalls the causes of Respiratory acidosis. Which of the following are
causes of acute respiratory acidosis?

1. Chronic obstructive pulmonary disease (COPD)


2. Pneumonia
3. Pulmonary edema
4. Atelectasis
5. Bronchitis

A. 1, 2, 3, 4, 5
B. 2, 3, 4, 5
C. 1, 3, 4, 5
D. 1, 2, 3, 4

52. Nurse Clara further recalls that respiratory acidosis nearly always results from
which of the following conditions?

A. Hyperventilation
B. Hypoventilation
C. Decrease amount of acid in body fluids
D. Low production of carbon dioxide

53. Nurse Clara reviews the results of the arterial blood gases. Which of the following
would the nurse expect to read in the report?

A. pH 7.25, PCO2 50 mm Hg
B. pH 7.40, PC02 52 mm Hg
C. pH 7.35, PC02 40 mm Hg
D. pH 7.50, PC02 30 mm Hg

54. The patient is worried about his kidneys being affected because of his condition.
The nurse explains that usually kidneys _________.

A. can achieve optimal compensation in about 72 hours


B. can achieve optimal compensation immediately
C. will compensate within 24 hours
D. are unable to compensate

55. Based on assessment data gathered, Nurse Clara writes a nursing diagnosis. Which
of the following is the MOST appropriate nursing diagnosis?
A. Risk for injury
B. Ineffective breathing pattern
C. Risk for respiratory infection
D. Ineffective tissue perfusion

Situation - Ms. E.D., 45-year old is admitted to the medical ward because of complaints
of muscle weakness, fatigue, ptosis and diplopia. The admitting diagnosis is myasthenia
gravis.

56. Based on the complaints of the patient, the nurse formulates a nursing diagnosis.
Which of the following is MOST appropriate?

A. Activity intolerance related to muscle weakness and fatigue


B. Imbalance nutrition: less than body requirements related to muscle weakness and
dysphgia
C. Ineffective airway clearance related to chest muscle weakness and impaired cough
and gag reflex.
D. Ineffective breathing patterns related to weakness of chest muscle and fatigue

57. The nurse administers anticholinesterase medication. When is the BEST time to give
the medication?

A. 30 minutes before meals


B. Before the patient sleeps at night
C. Early in the morning
D. When the patient has eaten a full meal

58. The nurse observes that the client had not been compliant with her medication
regimen of Pyridostigmine (Mestinon). The patient missed several doses. Which of the
following complications should the nurse watch for?

A. Gastrointestinal symptoms
B. Respiratory distress
C. Bradycardia
D. Vertigo

59. The nurse prepares Ms. E.D. for diagnostic test. Which of the following is NOT a
diagnostic for myasthenia gravis.

A. Tensilon test
B. Position Emission Tomography (PET)
C. Serum assay for circulating Ach receptor antibodies
D. Electromyography (EMG)

60. The nurse should always keep which of these drugs at the bedside of a client with
myasthenia gravis.

A. Atropine
B. Tensilon
C. Inderal
D. Neostigmine

Situation - You are a new registered nurse in the surgical unit. You admit a 30 year old
male with head injury sustained in a motor cycle accident. You understand that
reporting information is a critical part of documentation.

61. You are aware that documentation should reflect objective data. Based on your
assessment of the neurological function of the patient's LOC. you chart the following
observations. Which of the following is an objective data?

A. Client appears confused.


B. Client looks lethargic.
C. Vital signs are stable.
D. Patient has a score of 3-4-3 on the Glasgow Coma Scale.
62. You assess the pupils of the patient and record your observations. Which of the
following is LEAST important to record?

A. Symmetry
B. Size
C. Color
D. Reaction to light.

63. Based on the information you gathered from the patient, you write a nursing
diagnosis. Which of the following is NOT a well-written nursing diagnosis?

A. Disturbed sensory perception related to cerebral injury.


B. Fatigue related to cerebral injury.
C. Acute confusion related to altered cerebral blood flow.
D. Lethargy related to non-specific cause.

64. Based on the nursing diagnosis, you record an evaluation of the outcomes. Which
outcome is NOT well-written? The patient will _______?

A. rest as needed
B. be conscious, oriented and will perform own self-care
C. have functional sensory status.
D. feel lethargic.

65. You are aware that the primary purpose(s) of documentation are the following:
EXCEPT: to ________.

A. allow the nurse to express his or her opinion on patient care.


B. provide legal protection for the nurse in case of lawsuit.
C. collect data to improve quality of nursing care.
D. communicate patient information to other members of the health team.

Situation – The nurse admits a male patient with complaints of epigastric pain, fatigue,
anorexia, weight loss and pain in the right upper quadrant. The probable diagnosis is
primary liver cancer.

66. The nurse prepares the patient for the test ordered by the physician. Which of the
following procedures would confirm the diagnosis of liver cancer?

A. Computed tomography (CT) scan


B. Abdominal ultrasound
C. Abdominal flat plate x-ray
D. Cholangiogram

67. The physician performs a liver biopsy on the patient. Which of the following
complications should the nurse monitor IMMEDIATELY after the procedure?

A. Nausea and vomiting


B. Abdominal cramping
C. Hemorrhage
D. Potential infection

68. The physician prescribes Adriamycin for the patient. Which of the following
considerations has the HIGHEST priority when preparing to administer the medication
to a patient with liver cancer? __________of the medication.

A. Metabolism
B. Necessity
C. Purpose
D. Frequency

69. Based on of the information gathered, the nurse writes a nursing diagnosis. Which
of the following is the PRIORITY nursing diagnosis for the patient?

A. Acute pain related to abdominal pressure


B. Risk for infection related to complications of liver biopsy
C. Knowledge deficit related to self-care and cancer risk prevention
D. Fear and anxiety related to actual or potential lifestyle changes

70. The nurse understands the two basic types of liver cancer which are primary and
secondary. Which of the following statements is TRUE regarding primary liver cancer?

A. Women experience more primary liver cancer than men.


B. It is more common in developed countries.
C. It is more common in the presence of chronic renal disease.
D. Prognosis is poor: there is <20percent survival rate.

Situation - A 55-year old male is admitted to the medical unit with a diagnosis
Myocardial Infarction (MI).He complains of difficulty of breathing, excessive sweating,
nausea and vomiting and chest pain.

71. The nurse performs pain assessment. Which of the following characteristics of pain
is manifested in MI?

1. The patient may experience crushing substernal pain.


2. Pain may radiate to the jaw, back and leftarm.
3. Pain may occur without cause, primarily early in the morning.
4. Pain is unrelieved by rest or nitroglycerin and is relieved only by opiods.
5. Pain lasts 30 minutes or longer.

A. 2, 3, 4
B. 3, 4, 5
C. 1, 3, 5
D. 1, 2, 3, 4, 5

72. The physician orders laboratory tests on the patient. Which of the following findings
would MOST concern the nurse?

A. Creatine kinase: (CK): 150 U/ml


B. Hematocrit (HCT) 42
C. Serum glucose: 100mg /dL
D. Erythrocyte sedimentation rate (ESR):10mm/h

73. The physician further orders an arterial blood gas measurement. The nurse obtains
the specimen. What is the MOST appropriate action of the nurse immediately after
obtaining the specimen? The nurse should _________.

A. obtain ice for the specimen


B. apply a sterile dressing to the site
C. apply direct pressure to the site
D. observe the site for hematoma formation

74. The nurse reviews the arterial blood gas results of the patient. The laboratory
report indicates a pH of 7.30. PC02 of 58mmHg, PO2 of 80 mmHg, and a HCO3 of
27mEq/L. Which acid-base disturbance is the patient experiencing?

A. Metabolic acidosis
B. Metabolic alkalosis
C. Respiratory acidosis
D. Respiratory alkalosis

75. The nurse administers medications as prescribed by the physician. The nurse
monitors the blood pressure closely after giving the medications. If the blood is
pressure less than 100 systolic or 25 mmHg lower than the previous reading, what is
the INITIAL action of the nurse?

A. Elevate the head of the bed and notify the physician.


B. Lower the head of the bed and notify the physician.
C. Take the BP three times, get the average and report to the physician.
D. Reassure the patient. Explain that his BP is normal in his condition.
Situation - You are a newly hired registered nurse in a tertiary hospital. You are
required to attend an orientation activity on legal implications of nurses actions.

76. You are aware that you need a license to practice in the Philippines. Licensure is
primarily required to protect which of the following?

A. The patients under her/his care.


B. The school where the nurse obtained his/her nursing education and training
C. The hospital where s/he is employed.
D. Nurses because they are vulnerable to lawsuits.

77. The facilitator discusses negligence and malpractice. Which of the following factors
is unique to malpractice?

1. There is a contractual relationship between the nurse and the patient.


2. An inappropriate care is an act of commission.
3. The patient is harmed as a result of care.
4. The action of the nurse did not meet standards of care.

A. 3 only
B. 3 & 4
C. 1 only
D. 1 & 2

78. The facilitator cites a situation. Nurse x used a medical equipment improperly which
harmed the patient. The nurse may be charged with ________:

A. Negligence
B. Assault
C. Malpractice
D. Battery

79. The facilitator gave an example of a nurse who gave the wrong medication to a
patient. The result was severe allergic reaction. The nurse could be sued for _________:

A. Battery
B. Malpractice
C. Negligence
D. Assault

80. The nurse could be sued for which of the following if s/he says to a patient, "If you
don't stop complaining, I will not allow you to see your family when they visit."

A. Battery
B. Assault
C. Libel
D. Negligence

Situation - Nurse Luisa is assigned in the coronary care unit of a tertiary hospital. She
reviews the cardiovascular system before caring for patients with heart diseases.

81. Given a set of statements regarding the physiology of the cardiovascular system,
which of the following statements is TRUE?

A. When a person has heart muscle disease, the heart muscles stretches as far as is
necessary in order to maintain good function.
B. The heart rate increases when the parasympathetic system is stimulated.
C. The QRS interval on the electrocardiogram represents the electrical impulses
passing through the ventricles.
D. When there is a decrease in stroke volume, the heart rate decreases.

82. Nurse Luisa collects data from a patient with primary diagnosis of heart failure. The
patient reports that he has experienced the following disorders. Which disorder does
NOT precipitate heart failure?

A. recent upper respiratory infections


B. thyroid disorders
C. nutritional anemia
D. peptic ulcer disease

83. Nurse Luisa has a patient admitted for palpitations and mild shortness of breath. An
electrocardiogram (ECG) was taken. The results revealed a normal P wave, P-R interval,
and QRS complex with a regular rhythm and rate of 108 beats per minute. Nurse Luisa
recognizes this cardiac dysrhythmia as _________:

A. Sinus dysrhythmia
B. Supraventricular tachycardia
C. Sinus tachycardia
D. Ventricular tachycardia

84. The electrical activity of the patient's heart is being continuously monitored.
Suddenly the patient has a short burst of ventricular tachycardia followed by
ventricular fibrillation. Nurse Luisa should IMMEDIATELY ___________:

A. Run to the nurse's station quickly and call a code.


B. Administer atropine as ordered.
C. Prepare the patient for surgical placement of a pacemaker.
D. Call for help and initiate cardiopulmonary resuscitation.

85. Nurse Luisa attends to a patient who has continuous ECG monitoring. She observes
that the monitor shows that the rhythm has changed to ventricular tachycardia. Which
of the following interventions is the FIRST action by the nurse?

A. Quickly assess the level of consciousness, blood pressure, and pulse.


B. Administer a precordial thump.
C. Administer intravenous lidocaine following emergency protocol.
D. Quickly obtain a defibrillator and defibrillate the patient.

Situation - A 61 year old male seeks consultation in the OPD. He complains of blood in
the urine, pain on urination and frequent urination. Medical diagnosis is bladder cancer.

86. The male client is admitted to the oncology unit. A cystostomy is performed, a tumor
is visualized and biopsied. The nurse recognizes that the activity most often associated
with bladder tumors is _______:

A. Drinking three cans of carbonated beverages every day.


B. Smoking two packs of cigarettes a day.
C. Jogging 6 km a day.
D. working with a heavy equipment everyday

87. The client receives a radiation implant for the treatment of bladder cancer. Which of
the following intervention is appropriate?

A. Place client in isolation.


B. Encourage fluid intake.
C. Monitor client for signs and symptoms of cystitis.
D. Restrict fluid intake.

88. The physician plans to do a cystectomy and ideal conduit on the male client. The
nurse prepares the client for the procedure. Which of the following is an appropriate
action of the nurse?

A. Perform cleansing enema and give laxatives as ordered.


B. Teach the client muscle-tightening exercises.
C. Demonstrate to the client the procedure for irrigating the stoma.
D. Limit fluid intake for 24 hours.

89. The client undergoes a radical cystectomy and has conduit. Which of the following
postoperative assessment findings should the nurse observe and report to the physician
immediately?

A. Slight observe bleeding from the stoma when changing the appliance.
B. A urine output of more than 30 mL per hour
C. A red moist stoma.
D. A dusky, colored stoma

90. Which of the following instructions should the nurse give to the client with an ideal
conduit skin care at the stoma site?

A. Clean the skin around the stoma with mild soap and water and dry the area
thoroughly.
B. Leave the stoma open to air while changing the appliance.
C. Cut the face plate or wafer of the appliance no more than 4mm layer than stoma.
D. Change the appliance before going to sleep.

Situation - According to Peter Drucker "Management is doing things right" Resource


management is the process by which manager manage their resources effectively.
These resources can be intangible (people and time) and tangible (equipment, materials
and finances).It involves planning so that right resources are assigned to the right
tasks.

91. Currently, patients in the unit are bathed in the morning between 8 to 11 in the
morning. The nursing staff is considering changing the schedule to bathing the patients
in the evening between 7 to 9 p. m. Which statement reflects an assumption that might
affect the nursing staff's ability to think critically about the change?

1. “The day shift just wants to shift work onto the evening shift.”
2. “Everyone likes to bathe right before bed.”
3. “If we let them change this, they will want to make lots of other changes.”
4. “I read a research article that reported that clients sleep better if they have been
bathed right before bedtime.”

A. 3, 4
B. 2, 3, 4
C. 1, 2, 3
D. 1, 2

92. A serious disagreement has arisen between two staff nurses. The head nurse choose
not to make decision regarding the disagreement until more evidence is collected.
Which part of the nursing process does this head nurse's critical thinking reflect?

A. planning
B. Diagnosis
C. evaluation
D. assessment

93. A nurse supervisor who is retiring is helping the new supervisor learn about the
position, A part of this education the current supervisor make all the decisions that can
be made under certainty. What conditions will these decisions have in common?

1. There is risk associated in decision.


2. The alternatives are known
3. The conditions of each alternative are clear.
4. The decision is complex and dynamic.

A. 2, 4
B. 1, 2
C. 2, 3
D. 3, 4

94. A disagreement has arisen between two staff nurses. Both have discussed the
situation with the head nurse. The head nurse feels that the problem is likely sell
solving. How should the head nurse approach this situation?

1. Allow sometime to pass to see if the situation resolves


2. Support both nurses as they work thru issue.
3. Provide any resources the nurses may need to help solve the problem.
4. Intervene if the problem begins affect client care.
A. 1, 2, 3, 4
B. 1, 2, 3
C. 1, 4
D. 2, 3, 4

95. Which statement by a formal leader of a surgical unit reflects the personality
required to energize the staff and promote creativity?

1. “This is the decision I have made, and there is no room for discussion.”
2. “I would like to thank everyone for you hard work and dedication while we have been
short staffed.”
3. “I am certain that our new assignment system is going to make workload more
equitable.”
4. “I know this week has been rough”

A. 3, 4
B. 2, 3, 4
C. 1, 2
D. 1, 2, 3

Situation - Nurse Lory is assigned in the oncology unit of x hospital. She assists in the
care of a 40 year old female with bone cancer.

96. The patient complains of pain. Nurse Lory assesses the patient. Which of the
following statements is a MOST important information Lory gathers during the initial
assessment? The

A. patient's self-reporting of her pain experience


B. results of assessment of the physical examination
C. amount of pain medication the patient is taking
D. response of the family toward the illness of the patient

97. The patient informs nurse Lory that she is taking medications to control the pain.
Which of the following statements verbalized by the patient Indicates that the patient
needs further teachings about medications to control pain?

A. “It is okay to take my pain medication even if I am not having any pain.”
B. “I should take my pain medication periodically so I don't get addicted to the drug.”
C. “I should contact the nurse if the pain is not effectively controlled by my medication.”
D. “I should take my medications around-the-clock to control the pain.”

98. The patient receives chemotherapy: Nurse Lory writes a nursing diagnosis for the
patient. Which of the following nursing diagnoses is MOST appropriate

A. Altered body image


B. Pain related to treatment
c. Impaired physical mobility
D. Risk for infection

99. Nurse Lory is aware that a patient receiving chemotherapy is at risk for bone
marrow depression. She instructs the patient on how to prevent infection at home when
she is discharged. Which of the following should nurse Lory communicate to the
patient?

A. “Wash your hands frequently and maintain good hygiene.”


B. “Avoid physical contact with other people while receiving chemotherapy.”
C. “Do not share the bathroom with young children or with any pregnant member of the
family.”
D. “Visit the laboratory every week for a WBC analysis.”

100. Nurse Lory observes that the patient gets irritable and angry with the
medical/nursing staff whenever a procedure or treatment is done on her. Which of the
following approaches would be BEST to diffuse the anger of the patient?

A. Let the patient and family members have time for each other.
B. Direct the discussion and allow the patient to express her feelings.
C. Arrange a meeting between the patient and another person with bone cancer.
D. Request the social worker of the psychiatrist in the hospital to talk to the patient.

NURSING PRACTICE V

Situation – Nursing has a distinct and vital role in the management of injuries. The
prevention of sports injury is key to both sports management and health promotion in
sports.

1. Sancho, an eighteen years old college student is into sports. While playing football,
he sustained injury of the left knee. Nurse Sheila is assessing Sancho. Which of the
following signs and symptoms observed by Sheila reflect positive sign of soft tissue
injuries?

A. Pain, bleeding and swelling.


B. Sprain, fracture and severe pain
C. Pain, Swelling, redness and warmth
D. Bleeding, dislocation and pain

2. Nurse Sheila should include which of the following approaches in addressing the
initial injury management?

A. Relative rest, compression and elevation


B. Relative rest, ice, compression and elevation
C. Relative rest, protection, elevation and pain alleviation
D. Relative rest, elevation, pain elevation and ice

3. One of the most common quoted acronym in the sports injury literature for the initial
management is that of “DO NOT HARM”. This useful approach helps Nurse Sheila in
advising patients to avoid which of the following approaches:

1. Heat application to the affected area;


2. Alcohol Intake
3. Rest
4. Massage

A. 1, 2, 3 and 4
B. 1, 2 and 4
C. 1 and 2
D. 3 and 4

4. After the initial injury management, Nurse Sheila’s PRIORITY in her plan of care for
Sancho includes which of the following?

A. Restore optimal functional ability


B. Facilitate Sancho’s desire to return to sports
C. Alleviation of pain
D. Administration of oral non-steroidal anti-inflammatory drugs as ordered
5. Nurse Sheila knows that ice or cold therapy is commonly used during the acute stage
of sports injury. The following methods when applied correctly benefits that patient,
EXCEPT

A. One hour application to the injured part at a time continuously


B. Application should be done four to eight times daily.
C. Melting iced water through a wet towel
D. Ten minutes application to the injured part at a time and repeated as necessary

Situation – Nurses Melody takes care of her 80-year-old mother and 85years old father.
She noticed her parents to be forgetting the names of their long time house help and
repeatedly ask the same questions several times.

6. This condition has difficulty in processing stimuli and new information. Alzheimer’s is
the most common form of which of these?

A. Fatigue
B. Psychosis
C. Malnutrition
D. Dementia

7. Physiologically, what happens to the brain as Alzheimer’s progresses?

A. Brain stem atrophies


B. Cells die
C. Fluids Collect
D. Tissues swells

8. No drugs cure this condition. Emphasis is put on delaying the onset of severe
symptoms. Which of the following does NOT help improve Alzheimers?

A. Smoking
B. Eat balanced diet
C. Social connection and mental inactivity
D. Exercise

9. Which of these measures will help stabilize mentally the parents of Nurse Melody?

A. Establish a regular routine


B. Move to small apartment
C. Repaint or buy a new furniture.
D. Correct bad behaviors gently

10. Her parents usually becomes anxious and confused during late in the afternoon and
after dark. What do you call this phenomenon?

A. Agitation
B. Dark retreat
C. Sundowning
D. Dark reaction

Situation – Celestina, a 72-year old female, was seen wandering around Ayala Park
since early morning. At about noon she was found unconscious along the sidewalk. She
was brought to the Emergency Room of the nearest hospital. When she regained
consciousness, she was confused and cannot give any information about herself. She
was diagnosed with Delirium.

11. Upon admission, Celestina was perspiring profusely with a temperature of 103.2
degrees F and showing signs of severe dehydration. She was placed on I.V therapy. In
her case, what is the CAUSE of the Delirium?

A. Intoxication
B. Hypoglycemia
C. Severe dehydration
D. Vitamin C deficiency
12. Which of the following is the PRIMARY and often the INITIAL sign of Delirium?

A. Aphasia
B. Sleep disturbance
C. Altered level of consciousness
D. Apraxia

13. In managing clients with Delirium, it is important to give realistic reassuring to


them. Which of the following statements is the MOST APPROPRIATE for this nursing
actions?

A. “Stop crying Celestina. It will not make you any better.”


B. “I know things are upsetting and confusing right now. Your confusion should clear if
you get better.”
C. “Do not worry. Your confusion is normal at your age.”
D. “I know you are confused right now. Just wait for your medication to take effect.”

14. Celestina attempted to remove the I.V tubing from her arm, telling you “Get off me!
Go away!” She is experiencing which of the following?

A. Auditory hallucination
B. Disorientation
C. Visual hallucination
D. Delusions

15. Celestina gets agitated shouting, “There is a snake! There is a snake!”. Pointing to
the connecting cord of the electric fan. What would be your MOST APPROPRIATE
response?

A. “Celestina, you may hold it. You see, it is just a connecting cord.”
B. “Celestina, this is the connecting cord of the electric fan.”
C. “Celestina, where is it? I do not see any.”
D. “Celestina, it is just your imagination.”

Situation – Knowledge of legal terms, concepts and accountability is very important to a


professional nurse. These serve as guide for her personal as well as professional life.

16. You are assigned in the ICU of the Neurological Unit of the Hospital. ”The attending
physician ordered a one to one nursing care for his CVA patient. This means that the
nurse ______.

A. Can leave the patient’s bedside for a maximum of ten minutes


B. Must remain at the patient’s bedside until relieved by another one to one nurse
C. Can leave the patient’s bedside to do personal things only
D. Must leave the patient’s to restock the medicine supply

17. A sentinel events for a fractured patient undergoing hip replacement occurred in
the Operating Room. The surgeon may be held liable for the incident under which of the
following legal doctrines?

A. Respondent Superior
B. Force majeure
C. Captain of the ship
D. Il flagrance derelict

18. You are a staff nurse in a government hospital. An administrative case was filed
against you. Which of the following government agencies prescribes the mechanisms
for the investigation and punishment for your case?

A. Professional Regulation Commission


B. Ombudsman
C. Civil Court
D. Civil Service Commission

19. You are a staff nurse in a private hospital. Your co-worker spreads malicious rumors
against you. Which of the following agencies will you file the case of unprofessional
behavior against your co-worker?
A. Philippines Nurses Association
B. Civil Service Commission
C. Department of Labor
D. Professional Regulatory Board of Nursing

20. You are staff nurse in the Neurology Unit of the Hospital. You failed to report to
duty due to typhoon Yolanda that hit your place causing massive destruction. Whatever
sentinel event that may happen at the time of your absence, you may be exonerated
from any liability under the legal doctrine of _____.

A. Captain of the ship


B. Force Majeure
C. Justice
D. Respect the person

Situation – Research competency is expected of every professional nurse. The following


questions refer to this competency.

21. You are community health nurse is Municipality T. You want to determine the
effectiveness of the new program in Barangay Poblacion on listening technique for
preschoolers who have hearing problems. There are 2030 preschoolers in the barangay.
Using Slovin's formula with .05 margin of error. Your sample size will be ______.

A. 232
B. 350
C. 341
D. 250

22. To ensure the generalizability of research findings, you will utilize which of the
following sampling techniques?

A. Purposive sampling
B. Stratified random sampling
C. Convenience sampling
D. Simple random sampling

23. You want to find out the link between heavy smoking and cataract development.
Which of the following research designs are you going to use to answer the research
question?

A. Comparative
B. Prospective cohort
C. Retrospective cohort
D. Experimental

24. In a study on the relationship between smoking, and cataract development, an r=.3
would mean _______.

A. High positive correlation


B. Low negative correlation
C. medium negative correlation
D. low positive correlation

25. In a study on Alzheimer's Disease, the hypothesis offered was, "Nursing home
residents aged 65-90 years old who have Alzheimer's Disease die at an early age than
those who do not have". IDENTIFY the dependent variable.

A. Age at death
B. Living in a nursing home
C. Dying at an early age
D. Having Alzheimer's Disease

Situation - In interdisciplinary care of a psychiatric patient, each member of the health


team has his/her roles and functions. The following questions relate to this statement.
26. Which of the following is NOT an ingredient of the collaboration process?

A. Receptivity and respect for each health team member's contribution.


B. Active member and assertive contribution from each health team member
C. Negotiations between and among health team members to conceptualize new ways
of solving problems
D. Communication techniques used by each health team member

27. The MOST IMPORTANT step in interdisciplinary team building is to ______.

A. specify conflict resolution procedures


B. use disciplinary treatment plan
C. communicate regularly, openly, and clearly
D. define the disciplinary and individual roles and responsibilities

28. Which of the following is NOT a BARRIER to interdisciplinary collaboration?

A. Inequality of power and status


B. Personal qualities of the individual health care team member that promote shared
problem-solving
C. Role conflict
D. Inappropriate education and training of mental health team members

29. The element of collaboration essential in developing psychiatric nursing further is

A. Utilization of evidence-based interventions


B. To link nursing theory and clinical practice through research
C. Collaborative practice
D. The use of information

30. The mental health team member responsible for helping patients to received the
needed service is the

A. Psychiatric nurse
B. Social worker
C. Case worker
D. Psychologist

Situation – Teresa, a 30-year old female bank teller, was admitted to the Psychiatric
Unit of the hospital for depression and suicidal tendencies. You are the nurse assigned
to her.

31. You prepare a care plan for Teresa. Which of the following nursing care objectives is
given HIGHEST PRIORITY?

A. Maintain a calm environment to enable Teresa to express her feelings and thoughts
B. Use measures to protect Teresa from harming herself
C. Provide for contact between Teresa and her family
D. Reassure Teresa of her worthiness in a gentle manner

32. You formulate this nursing diagnosis for Teresa. “Potential for self-directed violence.
What would be your PRIORITY nursing action?

A. Let Teresa agree to sign a no-harm contract


B. Remove all potentially harmful objects from the environment
C. Assign Teresa to a double room with another patient
D. Instruct Teresa to call any nursing staff when she has thoughts of harming herself

33. Generally, it is difficult for you to maintain effective relationship with a depressed
patient like Teresa, who experienced suicidal ideation because of her

A. Laziness which keeps her from putting much effort to get well
B. Independence which prevents her from asking assistance from others
C. Pessimism which causes frustration and anger in others
D. Poor personal grooming which invites disgust and ridicule from others
34. You know that the danger of a suicide attempt is GREATEST when Teresa’s behavior
indicates that she

A. Agrees to visit with an estranged sister


B. Has an increased energy level
C. Is at a point of deepest despair
D. Has resumed her usual lifestyle

35. You evaluate the treatment done on Teresa. Which of the following behaviors
indicates that progress is being made? Teresa shows an improvement in her

A. Appetite
B. Self-concept
C. Activity level
D. Sleeping pattern

Situation - You are in a community health nurse in Municipality T. resource speaker in a


community assembly to address the growing incidence of alcoholism among the
students in the two public high schools of the municipality.

36. Being a resource speaker on a community health issue is part of your role and a/an
_____.

A. event coordinator
B. health educator
C. researcher
D. advocate

37. A student asks you on the risk factors of alcoholism among high school students.
Using a psychoanalytic theory as your frame of reference. Your answer is that
alcoholism is a fixation on what stage of a child's development?

A. Pre-school stage
B. Anal stage
C. Oral stage
D. School-age stage

38. From a behavioral perspective, you explain that the risk factor for alcoholism is
______.

A. due to low esteem of the teenagers


B. a learned, maladaptive behavior
C. a result of sexual abuse in the early stage of development
D. a fixation to the anal stage of development

39. A father, who is a known alcoholic, asks about the possibility of his son being an
alcoholic also. Your answer would be that the son _________.

A. will not develop alcoholism as it is not hereditary


B. has a lower risk for developing alcoholism
C. would develop alcoholism
D. has a higher risk for developing substance abuse to alcohol

40. A housewife whose alcoholic husband is undergoing detoxification process for


alcohol asks the importance of increasing fluids during this time. Which of the following
would be your MOST APPROPRIATE response? Fluids ______.

A. help remove alcohol from the body


B. prevent gastro-intestinal complications
C. prevent bladder complications
D. help prevent dehydration because alcohol is a diuretic

Situation - Nurse Arlene is caring for a patient who just had an intracerebral
hemorrhagic stroke. The family members are concerned about how this could happen.
41. Without knowing the cause, Nurse Arlene explains to the family which of the
following as the number one reason for intracerebral hemorrhagic stroke ?

A. Heparin use
B. Tachycardia
C. Fall
D. Chronic hypertension

42. With the diagnosis of hemorrhagic stroke, she teaches the caregiver that the
position of the bed should be in what position?

A. high fowlers
B. fowlers
C. trendelenberg
D. flat

43. Nurse Arlene is caring for another patient who is now recovering from stroke. The
attending physician ordered for a procedure to test his swallowing reflex. The son
wanted to know the purpose of such test. Which of the following best describe Nurse
Arlene's answer?

A. to determine the nutrients needed to put in TPN


B. to determine the type of feeding tube that is most appropriate
C. to assess the amount of dysphagia present
D. to check whether the patient has a medical condition that causing swallowing
problems

44. Nurse Arlene noticed that the patient still has difficulty in swallowing. His son was
told by Nurse Arlene that the patient is still on clear liquids. Which of the following will
Nurse Arlene EXCLUDE in her health teachings as examples of clear liquids?

A. Clear juice
B. Broth
C. Milk
D. Tea

45. Which of the following is indicative that the patient has fully recovered from stroke?

A. Burning and tingling sensation


B. Balance problem
C. Confusion
D. Presence of swallowing reflex

Situation – Lino, a 45-year old house painter came in with the chief complaints of
blurring vision, slurred speech, and hallucinations. Further evaluation of his case
revealed Volatile Substance Intoxication.

46. The predisposing factor in Lino’s case is his

A. Community
B. Occupation
C. Home environment
D. Age

47. Volatile Substance Abuse is considered the most dangerous among the abused
psychoactive substance because of the RISK of

A. Developing schizophrenia
B. Violence
C. Irreversible damage to the bone marrow, brain, liver, and kidney
D. Malnutrition

48. You heard Lino saying, “My mother visited me last night. She reminded me to take
care of myself”. This is a manifestation of

A. Reaction formation
B. Auditory hallucination
C. Visual hallucination
D. Delusion

49. Severe intoxication to volatile substances may lead to unconsciousness or even


death. The PRIORITY nursing action in caring for such patient is monitoring the

A. Mental status
B. Neurological functioning
C. Vital signs
D. Nutritional status

50. Substance abuse affects not only the user but also the other member of the family.
Which of the following is the MOST APPROPRIATE nursing diagnosis in the care of the
patient?

A. Impaired parenting
B. Dysfunctional family processes
C. Ineffective coping
D. Impaired social interaction

Situation – Stress and coping strategies are two important concepts in the care of
clients with maladaptive behaviors. The following questions relate to this statement.

51. Nurse Lizbeth is assigned in the long-term unit. In the last six months, she had
several patients who died. She had been observed manifesting ineffective coping. An
indication of this behavior is Lizbeth

A. Talking seriously with her partner about the deaths


B. Offering to work extra shifts for several weeks
C. Scheduling a group session with the hospital chaplain to discuss the deaths
D. Keeping busy with other activities and does not think about the deaths for several
days.

52. Mr. Pal has just been informed that he has Cancer of the Prostate. He assured his
nurse, Ms. Norma, that he is fine when he is not. The most physical evidence to Ms.
Norma indicative of Mr. Pal's feeling stressed is ______.

A. dilated, peripheral blood vessels


B. hyperventilation
C. decreased heart rate
D. constricted pupil

53. Nurse Melanie has recently chaged job to work with young adults. She recognizes
that the common sources of stress to this population group include, which of the
following?

1. marriage;
2. aging parents;
3. starting a new job;
4. leaving the parental home:
5. decreased physical abilities;
6. changing body structure

(SELECT ALL THAT APPLY)

A. 2, 4, 5
B. 2, 3, 6
C. 1, 3, 4
D. 1, 2, 5

54. Mr. Joseph, a 45-year-old account executive, is experiencing job-related stress


associated with fear of being laid off. This led him accepting projects requiring a deal of
travel. The MOST IMPORTANT health promotion strategy for Mr. Joseph would be
________.
A. time management
B. sleep
C. nutrition
D. exercise

55. When Nurse Calixa enters the room, Patient Wilma is on the phone. Suddenly,
Wilma slams down the phone, sweeps everything off the over bed table and demands
that Nurse Calix performed a several duties at that moment. The MOST APPROPRIATE
response of Nurse Calixa would be to _______.

A. provide Wilma the needed care calmly and quietly


B. allow Wilma to complete venting then respond calmly
C. Tell Wilma, "I will return later". then leave the room
D. warn Wilma that no care would be given her until the screaming ends

Situation - You are a staff nurse in the Psychiatric Unit. A depressed patient assigned to
you fell from her bed. Your head nurse asked you to submit an, Incident Report (IR).

56. The purpose why your head nurse asked you to submit an IR is, to _______.

A. note patterns of incidences in the same unit


B. place it in your 201 file
C. recall immediately the incidence that may result in an injury or damages
D. evaluate your performance

57. In writing the IR, which of the following is NOT included?

A. What actually happened


B. Who witnessed the incident
C. Medications given to the patient
D. Who was/were involved

58. What guideline is IMPORTANT in relation to IR?

A. It is filed in the Records Section of the hospital


B. No reference to the IR is placed in the 201 file
C. No reference to the IR is made in the patient’s chart
D. The incident in tiled in the nurse’s 201 file

59. Which of the following would prove that the nursing action the nurse carried out
met the standards of care?

A. Following the physician’s order


B. Documenting the procedures done
C. Doing the physical assessment
D. Utilizing the nursing process in providing safe, quality nursing care

60. A tool to help the nurse recall an incident she is involved in is___

A. Process recording
B. Incident report
C. Nurse's notes in the patient's chart
D. Anecdotal record

Situation - Ana is a staff nurse in the adolescent clinic of the Pediatric Department. One
of her patients is Sharon, a senior high school student, who is being treated for her
Bulimia.

61. Bulimia is BEST DEFINED as a/an____

A. eating disorder associated with excessive vomiting


B. disorder of unknown origin associated with starving onesel.
C. phobic disorder of fear of obesity
D. pathological disorder of binging and vomiting

62. What condition is NOT likely to develop in Sharon?


A. Dental erosion
B. Gastric ulcers
C. Rectal bleeding
D. Hyperkalemia

63. Endocrine changes often results in a bulimic patient. Which of the following would
be an expected change in Sharon?

A. Increased production of Follicle-Stimulating Hormone


B. Delayed Thyroid Stimulating Hormone response to Hormone Replacement Therapy
C. Decreased Adrenocorticotropic Hormone in response to cortisone
D. Hypopituitarism

64. Which of the following conditions may lead to death in a bulimic patient?

A. Hyponatremia and circulatory collapse


B. Hypokalemia and cardiac arrhythmias and arrest
C. Hypernatremia and congestive heart failure
D. Metabolic acidosis and renal failure

65. Amitriptyline (Elavil), an Anti-depressant, is the drug of choice in treating Bulimia.


What is a COMMON side effect of this drug?

A. Urinary frequency
B. Diarrhea
C. Anticholinergic effects
D. Cholinergic effects

Situation – Nurse Letty, a clinic nurse of ENT specialist receives patients with cataract
and glaucoma cases. She first conducts her initial nursing assessment before referring
it to DR. Kuan.

66. Nurse Letty is preparing to test the visual acuity of a client using a Snellen chart.
Which of the following identifies the accurate procedure for this visual acuity test?

A. The client is asked to stand at a distance of 40ft. from the chart and is asked to read
the largest line on the chart.
B. Both eyes are assessed together, followed by the assessment of the right and then
the left eye.
C. The client is asked to stand at a distance of 40ft from the chart and to read the line
that can be read 200ft away by an individual with unimpaired vision.
D. The right eye is tested to followed by the left eye, and then both eyes are tested.

67. The client’s vision is tested with a Snellen’s chart. The results of the tests are
documented as 20/60. The nurse interprets this as __________.

A. The client is legally blind


B. The client can read at a distance of 60 feet what a client with normal vision can read
at 20 feet
C. The client can read only at a distance of 20 feet what a client with normal vision can
read at 60 feet
D. The client’s vision is normal

68. Tonometry is performed on the client with a suspected diagnosis of glaucoma. The
nurse analyses the test results as documented in the client’s chart and understands that
normal intraocular pressure is ________ mmHg.

A. 22-30
B. 31-35
C. 10-21
D. 2-7

69. The nurse is developing a plan of care for the client scheduled for cataract surgery.
The nurse documents which more appropriate nursing diagnosis in the plan of care?
A. Disturbed sensory perception
B. Self-care deficit
C. Anxiety
D. Imbalanced nutrition

70. The part of the ear that contains the receptors for hearing is the _____.

A. Tympanic cavity
B. Utricle
C. Middle ear
D. Cochlea

Situation – The practice of nursing requires rules and regulations to ensure patient
safety and competent level of behavior in the professional role as a nurse. The following
questions are related to this statement.

71. A nurse attends a Halloween party dressed in white nurse’ uniform. She becomes
drunk and begins talking about her colleagues in a demeaning manner. Which of the
following will the nurse be considered?

A. Not presenting a positive or professional image of a nurse


B. Appropriate for the situation as it was a private party anyway
C. A professional misconduct
D. Someone who has the right to vent regarding her colleagues behavior

72. Two nurses are talking about specific client in the hospital cafeteria. These nurses
are at risk of being accused of which of the following?

1. professional misconduct
2. gross incompetence
3. behavior unbecoming of the profession
4. breach of confidentiality

A. 1, 2
B. 3, 4
C. 1, 3, 4
D. 1, 2, 3

73. If a patient to be discharged suggests to the nurse that she would like to get
together again if she feels depressed. What should the nurse response be?

A. Agree to meet with the patient at her convenience


B. Provide the patient with information about crisis center
C. It’s okay as long as they meet in a public place
D. Suggest they go to the gym together to exercise

74. Keeping your promise as a nurse to a patient to return in 10 minutes develops


_______.

A. empathy
B. closure
C. sympathy
D. trust

75. A nurse observes a patient driving his car. The nurse knows the patient has seizure
disorder and his driver’s license has been suspended. What should the nurse do?

A. discuss the observation from the doctor


B. notify the police department of the observation
C. follow the patient home and call the police
D. call the patient and ask whether he has been driving

Situation – You are a staff nurse in the Out-Patient Department. Mrs. D. brought her
nine-year old daughter, Bianca for consultation. Bianca is complaining of sore throat,
muscle tenderness, weakness of the arms, and a general feeling of not being well. The
attending physician’s impression is Infectious Polyneuritis.
76. Initially, What assessment data would be the MOST IMPORTANT for you to make?

A. Exposure to an infectious disease


B. Difficulty in swallowing
C. Diet intake for the last 24 hours
D. Difficulty in urinating

77. Bianca was admitted to the Pediatric Ward. What would be your PRIORITY nursing
action when caring for her?

A. Assessing Bianca’s ability to follow simple commands


B. Making a game of range-of-motion exercises
C. Evaluating Bianca’s bilateral muscle strength
D. Providing a diversional activity for Bianca

78. The pediatric ward nurse asks Bianca to cough every now and then. She also
assesses Bianca’s speech for decreased volume and clarity. The underlying rationale for
these two assessment activities is to determine the _________.

A. Increased intracranial pressure


B. Inflammation of the larynx and epiglottis
C. Regression to an earlier developmental phase
D. Involvement of facial and cranial nerves

79. Assessment findings revealed that Bianca has weak gag and cough reflexes. During
this acute phase of Bianca’s illness, which of the following problems should receive the
highest PRIORITY?

A. Total urinary incontinence related to fluid losses


B. Impaired swallowing related to neuro-muscular impairment
C. Risk for infection due to an altered immune system
D. Ineffective breathing pattern related to neuromuscular impairment

80. Bianca was placed on a mechanical ventilator. Which of the following nursing
actions should the nurse give emphasis to?

A. Transferring Bianca to a bedside chair three times a day to prevent postural


hypotension
B. Maintaining Bianca in a supine position to prevent unnecessary nerve stimulation
C. Engaging Bianca in vigorous passive range-of-motion exercises to prevent loss of
muscle functions
D. Turning Bianca slowly and gently from side to side to prevent respiratory
complications

Situation – You are assigned in the Orthopedic Unit of the Surgical Ward. It has been
observed that incidence of infection is increasing among patients on skeletal traction.
As a basis of improving care of enough knowledge on their care

81. To reduce the risk of infection in patients on skeletal traction, which of the following
principles must be observed?

A. The skeletal traction must be applied under general anesthesia


B. Apply the skeletal traction under sterile condition
C. Apply the counter traction immediately after application of skeletal pins
D. Start the patient on antibiotics at least 24 hours before the procedure

82. A patient on skeletal traction has increased risk of infection. Which of the following
pin sites are considered to have a greater risk for infection?

A. Pins placed through the skin


B. Pins placed through the bones
C. Areas with considerable soft tissue
D. Areas within the line of pull
83. Pin site care is a most important nursing care for patients on skeletal traction to
reduce risk of infection. Which of the following is the most effective cleansing solution
for this purpose?

A. Hydrochloric acid solution


B. Chlorhexidine solution
C. Soapsuds solution
D. Acetic acid solution

84. Which of the following signs for a patient on skeletal traction should be reported to
the physician? SELECT ALL THAT APPLY

1. Redness;
2. Loss of appetite;
3. Drainage;
4. Increased tenderness;
5. Formation of renal calculi

A. 1, 2, 5,
B. 1, 3, 4
C. 1, 2, 3
D. 1, 3, 5

85. Skeletal traction causes increased anxiety and discomfort. Which of the following
nursing actions would provide indicators for increased anxiety and discomfort to a
patient on skeletal traction?

A. Maintain the line of pull


B. Do pin site care daily
C. Perform skin assessment frequently
D. Perform neurovascular assessments frequently

Situation – You are the Nurse assigned to a patient with Amyotropic Lateral Sclerosis is
a debilitating disease.

86. Which of the following is a primary consideration in planning for her care?

A. To prevent problems related to immobility


B. Planning for client’s eventual inability to communicate
C. To prevent infection
D. To prevent complications

87. Which of the following is the initial teaching given to the patient with ALS and her
family?

A. Referral to support group


B. Prevention of complications
C. Explaining the disease process, expected course and prognosis
D. Prevention of depression

88. Which of the following is the initial teaching given to the patient with ALS having
problems in communication?

A. Referring the client to a speech therapist


B. Documenting any change in the elimination functions
C. Use of pre-signals before the loss of speech
D. Continue to use therapeutic communication techniques

89. Rilutex is the only drug approved by the FDA for treatments of ALS. Which of the
following would you observe when administering the medication?

A. Avoid giving the medication when the client is in pain


B. Administer the medication on an empty stomach
C. Administer the medication two hours after meals
D. Administer the medication with food
90. Death in clients with ALS occurs within two to three years and is attributable to
____.

A. fatigue
B. respiratory failure
C. liver toxicity
D. malnutrition

Situation – Nurse Linda, a newly hired nurse was assigned in Poveda Mental Clinic took
care of different patients having problems with thoughts and perceptions

91. Mary is pacing, complaining of financial problems and repeatedly utters multiple
different lines. Nurse Linda asks the client if something upsetting happened and Mary’s
response is vague and not focused on the question. Nurse Linda assess Mary’s level of
anxiety as_________.

A. Panic
B. Severe
C. Moderate
D. Mild

92. Nurse Linda observes a client pacing in the hallway. Which statements by the nurse
may help the client recognize his anxiety?

A. “Have you been pacing for a long time?”


B. “I guess you’re worried about something, aren’t you?
C. “I notice that you’re pacing. How are you feeling?”
D. “Can I get you some medication to help calm you?”

93. An 83-year-old male client was at extended care facility. He is anxious most of the
time and frequently complains of a number of vague symptoms that interfere with his
ability to eat. These symptoms indicate which of the following disorders?

A. Hypochondriasis
B. Conversion disorder
C. Sublimation
D. Severe anxiety

94. Mark, with a diagnosis of generalized anxiety disorder, wants to stop taking his
Lorazepam (Ativan). Which of the following important facts should Nurse Linda discuss
with the client about discontinuing the medication? Stopping the drug__________.

A. increases cognitive abilities


B. decreases sleeping difficulties
C. can cause withdrawal symptoms
D. may cause depression

95. Nurse Linda is having a conversation with a depressed client in an inpatient


psychiatric unit. The client says to her “Things would be so much better for everyone if
I just weren’t around”. Which of the following would be the most appropriate
response?”

A. “You sound very unhappy. Are you thinking of harming yourself?”


B. “Those feelings will go away once your medication really takes effect.”
C. “Have you talked to anyone specifically about what is bothering you?”
D. “I know what you mean; everyone gets that way when they are depressed.”

Situation – The use of therapeutic touch is an effective communication techniques in the


care of psychiatric patients.

96. You do a physical assessment on a newly admitted patient in the Psychiatric Unit.
Which type of therapeutic touch is used to assess the patient’s skin turgor?

A. Social-polite touch
B. Friendship-warmth touch
C. Functional-professional
D. Love-intimacy touch

97. What type of therapeutic touch is used when you gently guide the patient in going
to her room?

A. Love-intimacy-touch
B. Friendship-warmth touch
C. Sexual-arousal touch
D. Social-polite touch

98. You put your arms around the shoulders of an elderly patient. Which type of touch is
this?

A. Love-intimacy touch
B. Social-polite touch
C. Friendship-warmth touch
D. Functional-professional

99. Family members use touch involving tight hugs and kisses. What type of touch is
this?

A. Sexual-arousal touch
B. Love-intimacy-touch
C. Social-polite touch
D. Friendship-warmth touch

100. Which of the following type of touch is NOT used by the nurse in providing care to
psychiatric patients?
A. Social-polite touch
B. Sexual-arousal touch
C. Friendship-warmth touch
D. Love-intimacy touch

Nothing is impossible.
The word itself says
‘I’m POSSIBLE’

-Audrey Hepburn

You might also like